[obm-l] Re: [obm-l] Somatório

2011-05-09 Por tôpico fabio henrique teixeira de souza
Note que i(i+1) = 2.[Combinação de i+1 escolhidos 2 a 2]

Em seguida, use uma das propriedades do Triângulo de Pascal-Tartaglia.

Em 9 de maio de 2011 14:17, Kleber Bastos klebe...@gmail.com escreveu:

 Olá Pessoal,

 Não esotu conseguindo fazer o seguinte exercício:

 Provar que somatório de i=1 a n de i(i+1)  é igual a [n(n+1)(n+2)]/3
 Alguém póderia ajudar?

 Abraços,

 --
 Bastos



Re: [obm-l] Descobrir formula geral e provar f(n+1)=2f(n) +3

2011-05-09 Por tôpico fabio henrique teixeira de souza
Considere a(n) uma solução de f(n+1) = 2f(n)
Há infinitas soluções para tal, mas a(n) sempre será uma PG de razão 2.
Assim, uma solução é a(n) = 1.2^(n-1)

Vamos promover a mudança de variável f(n) = g(n).a(n)

Assim,
f(n+1) = 2f(n) + 3 se transforma em

g(n+1).a(n+1) = 2.g(n).a(n) + 3
g(n+1).2^n = 2.g(n).2^(n-1) + 3
g(n+1).2^n = g(n).2^n + 3

 g(n+1) = g(n) + 3.2^(-n)

Portanto,

g(1) = g(0) + 3.1
 g(2) = g(1) + 3.(1/2)
 g(3) = g(2) + 3.(1/4)
...
g(n) = g(n-1) + 3.(1/2^(n-1))

Somando:

g(n) = g(0) + 3.(1 + 1/2 + 1/4 + ... + 1/2^(n-1))
g(n) = g(0) + 3.[1/2^n - 1]/[1/2 - 1]
g(n) = g(0) - 6.[1/2^n - 1]

Note que f(0) = g(0).a(0) == 0 = g(0).2^(-1) == g(0) = 0

g(n) = 6.[1 - 1/2^n]

g(n) = 6.[2^n - 1]/2^n

f(n) = a(n).g(n) = 2^(n-1) . 6.[2^n - 1]/2^n
f(n) = 6.[2^n - 1]/2
f(n) = 3.[2^n - 1]








































































































































































































































Em 6 de maio de 2011 12:43, Julio Teixeira jcesarp...@gmail.com escreveu:

 Pessoal, a um tempo acho que vi essa questao aki  e por acaso, ontem me
 deparei com ela em alguns foruns, e o pessoal estava com dificuldades..entao
 vou por aki a minha resolucao..

 questao 157 do Vol. 1 da colecao do G. Iezzi - Fundamentos de matematica
 elemtentar

 157 - Seja f uma funcao, definida no conjunto dos numeros naturais, tal
 que, f(n+1)=2f(n) +3
 com f(0) = 0. Achar a formula geral de f(n) e prova-la por inducao..

 equacao: f(n+1)=2f(n) + 3  e f(0)=0

 para..
 n=0 = f(0+1)=2f(0)+3 = f(1)=3
 n=1 = f(1+1)=2f(1)+3 = f(2)=9
 n=2 = f(2+1)=2f(2)+3 = f(3)=21
 n=3 = f(3+1)=2f(3)+3 = f(4)=45
 n=4 = f(4+1)=2f(4)+3 = f(5)=93

 observando os valores retornado pelas imagens e pondo em produto de um
 fator por 3..
 f(1)=3  = f(1)=3*1
 f(2)=9  = f(2)=3*3
 f(3)=21 = f(3)=3*7
 f(4)=45 = f(4)=3*15
 f(5)=93 = f(5)=3*31

 agora observando os segundos fatores dos produtos acima nas imagens...
 comecamos com 1, depois 3, depois 7, e

 assim temos:
 a diferenca entre  3 e 1 = 2
 a diferenca entre  7 e 3 = 4
 a diferenca entre 15 e 7 = 8
 a diferenca entre 31 e 15 = 16

 obrservando essas diferencas, nota-se que temos uma PG, de razao 2, e com o
 primeiro termo sendo igual a 1

 assim a formula ja comeca a ficar evidente.. sendo 3 vezes essas
 diferencas...

 agora se montarmos essa PG, teremos..

 a1 = 1
 a2 = 2
 a3 = 4
 a4 = 8
 a5 = 16

 opa.. entao a proxima observacao a ser feita eh que, com os resultados
 obtidos temos que,por exemplo,
 f(1)=3*( a1 de nossa PG)
 f(2)=3*( a soma de a1 com o a2 de nossa PG)
 f(3)=3*( a soma de a1 com o a2  e a3 de nossa PG)
 f(4)=3*( a soma de a1 com o a2  e a3 e a4 de nossa PG)

 agora a formula do somatorio de nossa PG seria:
 Sn = a1 * (q^n - 1)/ (q - 1)

 onde substituindo, obteriamos:
 2^n -1

 agora deduzimos entao que a formula geral seria: f(n)= 3 * ( 2^n - 1)

 para provarmos por inducao, vamos provar que eh valido para n=1
 f(1) = 3 * ( 2^1 -1)
 f(1) = 3 * ( 1 ) = f(1) = 3   ( OK, provamos para n=1 )

 agora substituimos por n, por um k, qualquer e obtemos:
 f(k)= 3 * (2^k -1)

 agora substituimos por k+1
 f(k+1)= 3 * (2^(k+1) -1)

 ok, agora note que se pegarmos a formula inicial e aplicarmos n=k,
 obteremos o seguinte..
 f(k+1)=2 * f(k) + 3

 ja que obtemos f(k+1) de nossa formula e f(k+1) da formula original, para
 provarmos que descobrimos a formula geral
 entao o resultado de f(k+1), tem que ser igual, assim tb testamos se eh
 valida para qualquer elemento, provando isso para qualquer sucessor de k, ou
 seja (k+1)
 entao temos o seguinte..
 f(k)= 3 * (2^k -1)
 f(k+1)= 3 * (2^(k+1) -1)
 f(k+1)=2 * f(k) + 3

 agora igualando os f(k+1), obtemos..
 2 * f(k) + 3 = 3 * (2^(k+1) -1)
 substituindo f(k), pelo valor conhecido tb.. ( da nossa formula geral )
 2 * (3 * (2^k -1)) + 3 = 3 * (2^(k+1) -1)
 6 * (2^k -1) + 3 = 3 * (2^(k+1)) -3
 agora, dividimos amobs os lados por 3
 2 * (2^k -1) + 1 = 2^(k+1) - 1
 2^(k+1) -2 + 1 = 2^(k+1) - 1
 2^(k+1) - 1 = 2^(k+1) - 1(OK)
 obtemos assim, a nossa prova...







Re: [obm-l] Fwd: Identidade de Euler (OFFTOPIC)

2011-05-02 Por tôpico fabio henrique teixeira de souza
Obrigado a todos pela ajuda.

Em 28 de abril de 2011 19:44, Carlos Nehab ne...@infolink.com.br escreveu:

 Oi, querido amigo.
 Grande abraço
 Nehab

 Em 28/4/2011 17:40, Carlos Victor escreveu:

 Oi  Mestre  Nehab ,
 Gostei da sugestão e mais ainda  das n pessoas que moram em Nilópolis (
 minha  terrinha).

 Abraços

 Carlos  Victor

 Em 28 de abril de 2011 17:21, Carlos Nehabne...@infolink.com.br
  escreveu:

 Oi, Fábio,

 Não resisti:

 Resolva os seguinte problema de duas maneiras (uma técnica básica e útil
 para resolver identidades deste tipo).
 De quantas maneira posso formar comissões de p pessoas, a partir de um
 total de m + n pessoas, sendo m o total de pessoas que moram no Maracanã
 e n
 as pessoas que moram em Nilópolis?

 Abraços,
 Nehab

 Em 28/4/2011 13:24, fabio henrique teixeira de souza escreveu:

 -- Mensagem encaminhada --

 De: fabio henrique teixeira de souzafabiodja...@ig.com.br
 Data: 28 de abril de 2011 08:52
 Assunto: Identidade de Euler
 Para: obm-l@mat.puc-rio.br


 Pessoal, estou batendo cabeça e não consigo demonstrar que
 C(m,0).C(n,p) + C(m,1).C(n,p-1) + C(m,2).C(n,p-2) + ... + C(m,p).C(n,0)
 =
 C(m+n,p)

 Alguém pode me dar uma dica?



 =
 Instruções para entrar na lista, sair da lista e usar a lista em
 http://www.mat.puc-rio.br/~obmlistas/obm-l.html
 =


 =
 Instruções para entrar na lista, sair da lista e usar a lista em
 http://www.mat.puc-rio.br/~obmlistas/obm-l.html
 =



[obm-l] Identidade de Euler

2011-05-01 Por tôpico fabio henrique teixeira de souza
Pessoal, estou batendo cabeça e não consigo demonstrar que
C(m,0).C(n,p) + C(m,1).C(n,p-1) + C(m,2).C(n,p-2) + ... + C(m,p).C(n,0) =
C(m+n,p)

Alguém pode me dar uma dica?


[obm-l] Fwd: Identidade de Euler

2011-04-28 Por tôpico fabio henrique teixeira de souza
-- Mensagem encaminhada --
De: fabio henrique teixeira de souza fabiodja...@ig.com.br
Data: 28 de abril de 2011 08:52
Assunto: Identidade de Euler
Para: obm-l@mat.puc-rio.br


Pessoal, estou batendo cabeça e não consigo demonstrar que
C(m,0).C(n,p) + C(m,1).C(n,p-1) + C(m,2).C(n,p-2) + ... + C(m,p).C(n,0) =
C(m+n,p)

Alguém pode me dar uma dica?


[obm-l] Re: [obm-l] Re: [obm-l] FRAÇÕES - conceito

2011-03-24 Por tôpico fabio henrique teixeira de souza
Ralph, obrigado.
Além de aprender com você, ainda me divirto.

EMMOSC (em minha modesta opinião sobre convenções):
- fração é exatamente o que diz a SMO;
- 0 é natural;
- futebol com jogadores de madeira é totó;
- a fruta é tangerina

Mas não, não vou encarar.
Até porque você é maior, mais velho e mais inteligente do que eu.

Forte abraço.

Fabio Henriqu


Em 23 de março de 2011 18:18, Ralph Teixeira ralp...@gmail.com escreveu:

 Minha resposta é diplomática -- depende do que você chamar de
 fração. Defina do seu jeito, que seja conveniente para o que você quer
 fazer, e deixe claro a todos o que você está fazendo. Depois, seja
 coerente.

 (Ou seja, enrolei enrolei e não respondi.)

 Em Minha Modestíssima Opinião, fração é qualquer expressão do tipo a/b
 onde a e b são números ou até mesmo outras expressões. Então 1/(raiz
 de 2) é uma fração tanto quando 7/1 ou 25/pi ou (x+cos(y))/(z+w^2). Eu
 também diria que 3 não é uma fração, mas pode ser escrito como 3/1,
 que é uma fração... para mim, 45.78 não é fração, mas PODE SER ESCRITO
 como uma fração, 4578/100.

 Mas isso tudo é EMMO... Não, minto, é EMMC (Em Minha Modestíssima
 Convenção). Poxa, EMMC, 0 é natural, 0^0=1, aquele futebol com
 jogadores de madeira é pebolim e aquela fruta é mixirica Não
 gostou? Vai encarar? :) :) :) :)

 Abraço,
 Ralph

 2011/3/21 fabio henrique teixeira de souza fabiodja...@ig.com.br:
  Senhores, 1/(raiz de 2) é uma fração?

 =
 Instruções para entrar na lista, sair da lista e usar a lista em
 http://www.mat.puc-rio.br/~obmlistas/obm-l.html
 =



[obm-l] FRAÇÕES - conceito

2011-03-21 Por tôpico fabio henrique teixeira de souza
Senhores, 1/(raiz de 2) é uma fração?


[obm-l] Concurso Pré Vestibular CEDERJ

2010-01-27 Por tôpico fabio henrique teixeira de souza
Peço aos amigos que divulguem no Estado do Rio de Janeiro:



Concurso para seleção de tutores do Pré Vestibular do CEDERJ.



Inscrições on-line até 31/01 para graduados e graduandos não só de
matemática mas de todas as disciplinas.



Informações em http://www.pvs.cederj.edu.br/professores/


Um abraço e obrigado.
Fabio Henrique


[obm-l] Re: [obm-l] Re: [obm-l] Re: [obm-l] Re: [obm-l] Ques tão de Probabilidade - CESPE

2009-01-24 Por tôpico fabio henrique teixeira de souza
Valeu. Um abraço.

2009/1/24 Marcelo Salhab Brogliato msbro...@gmail.com

 Olá Fábio,
 eita, realmente não são equiprováveis. Desculpe pela falha.. hehehe
 Acho que é só multiplicar as probabilidades dos casos favoráveis e somar:
 11 .. neste caso, temos: 1/2*1/2 = 1/4
 1011 .. neste caso, temos: 1/2*1/2*1/2*1/2 = 1/16

 logo, P = 1/4 + 1/16 = 5/16

 espero nao ter errado novamente ;) hehe
 mas desta vez bateu com sua resposta... acho que acertamos!! :D

 abraços,
 Salhab


 2009/1/23 fabio henrique teixeira de souza fabiodja...@ig.com.br

  Salhab, agradeço a ajuda. No entanto, acho que vc cometeu um engano. Não
 podemos calcular a chance de A ser campeão por 2 casos favoráveis em um
 total de 5, posto que os 5 não são equiprováveis. Estou errado?
 Abraço.

 2009/1/23 Marcelo Salhab Brogliato msbro...@gmail.com

 Olá Fábio,
 dado que A perdeu o primeiro jogo, temos as seguinte possiveis
 configuracoes para os demais jogos:
 vamos convencionar: 1 = A venceu, 0 = B venceu (ou A perdeu).
 0 (neste caso B é campeao)
 11 (neste caso A é campeao)
 100 (neste caso B é campeao)
 1010 (neste caso B é campeao)
 1011 (neste caso A é campeao)
 logo, temos 2/5 = 40% de chance de A ser campeão.

 Outra maneira seria calcular: P(A ser campeao | A perdeu a primeira
 partida) = P(A ser campeao e A perder a primeira partida)/P(A perder a
 primeira partida)

 P(A perder a primeira partida) = 1/2
 P(A ser campeao e A perder a primeira partida) = 2/10 [basta ver que
 temos 2 casos favoraveis em 10, pois temos que considerar os casos em que A
 ganhou a primeira partida, que sao analogos aos acima]

 assim: P = (2/10)/(1/2) = 2/5 = 40%

 acho que é isso!!
 abraços,
 Salhab




 2009/1/22 fabio henrique teixeira de souza fabiodja...@ig.com.br

  Senhores, ao fazer a questão abaixo, encontrei como probabilidade 5/16.
 De saída, o problema não deixa claro que as probabilidades de vitória de A 
 e
 de B são iguais. Considerei-as iguais (a 1/2). No entanto, o gabarito
 oficial diz que a declaração é CERTA. Gostaria que alguém também a fizesse
 para que eu possa ter idéia se estou errando ou acertando. Aguardo o
 retorno.




 Analise em certo ou errado:

 A etapa final de um torneio de futebol será disputada entre os

 times A e B, e o campeão será o time que vencer duas partidas

 seguidas ou um total de três partidas. Considerando que os jogos

 que terminarem empatados serão decididos nos pênaltis, de forma

 que sempre haja um vencedor, julgue os itens que se seguem.

 109
 A chance de o time A ser campeão tendo perdido o primeiro

 jogo é de 20%.







[obm-l] Re: [obm-l] Re: [obm-l] Questão de Probabilidade - CESPE

2009-01-23 Por tôpico fabio henrique teixeira de souza
Salhab, agradeço a ajuda. No entanto, acho que vc cometeu um engano. Não
podemos calcular a chance de A ser campeão por 2 casos favoráveis em um
total de 5, posto que os 5 não são equiprováveis. Estou errado?
Abraço.

2009/1/23 Marcelo Salhab Brogliato msbro...@gmail.com

 Olá Fábio,
 dado que A perdeu o primeiro jogo, temos as seguinte possiveis
 configuracoes para os demais jogos:
 vamos convencionar: 1 = A venceu, 0 = B venceu (ou A perdeu).
 0 (neste caso B é campeao)
 11 (neste caso A é campeao)
 100 (neste caso B é campeao)
 1010 (neste caso B é campeao)
 1011 (neste caso A é campeao)
 logo, temos 2/5 = 40% de chance de A ser campeão.

 Outra maneira seria calcular: P(A ser campeao | A perdeu a primeira
 partida) = P(A ser campeao e A perder a primeira partida)/P(A perder a
 primeira partida)

 P(A perder a primeira partida) = 1/2
 P(A ser campeao e A perder a primeira partida) = 2/10 [basta ver que temos
 2 casos favoraveis em 10, pois temos que considerar os casos em que A ganhou
 a primeira partida, que sao analogos aos acima]

 assim: P = (2/10)/(1/2) = 2/5 = 40%

 acho que é isso!!
 abraços,
 Salhab




 2009/1/22 fabio henrique teixeira de souza fabiodja...@ig.com.br

  Senhores, ao fazer a questão abaixo, encontrei como probabilidade 5/16.
 De saída, o problema não deixa claro que as probabilidades de vitória de A e
 de B são iguais. Considerei-as iguais (a 1/2). No entanto, o gabarito
 oficial diz que a declaração é CERTA. Gostaria que alguém também a fizesse
 para que eu possa ter idéia se estou errando ou acertando. Aguardo o
 retorno.




 Analise em certo ou errado:

 A etapa final de um torneio de futebol será disputada entre os

 times A e B, e o campeão será o time que vencer duas partidas

 seguidas ou um total de três partidas. Considerando que os jogos

 que terminarem empatados serão decididos nos pênaltis, de forma

 que sempre haja um vencedor, julgue os itens que se seguem.

 109
 A chance de o time A ser campeão tendo perdido o primeiro

 jogo é de 20%.





[obm-l] Re: [obm-l] Re: [obm-l] Re: [obm-l] Re: [obm-l] Ques tão de Probabilidade - CESPE

2009-01-23 Por tôpico fabio henrique teixeira de souza
Ralph, valeu pela moral e pela resposta.

2009/1/23 Ralph Teixeira ralp...@gmail.com

 Concordo com o Fábio. Não são equiprováveis é a pedra no sapato de
 98% dos problemas de probabilidade que dão errado... :)

 Então, usando o raciocínio do Marcelo, temos as seguintes opções (dada
 a primeira derrota de A, daqui para a frente):
  0 (neste caso B é campeao) com probabilidade 1/2
  11 (neste caso A é campeao) com probabilidade 1/2.1/2=1/4
  100 (neste caso B é campeao) com probabilidade 1/2.1/2.1/2=1/8
  1010 (neste caso B é campeao) com probabilidade 1/2.1/2.1/2.1/2=1/16
  1011 (neste caso A é campeao) com probabilidade 1/2.1/2.1/2.1/2=1/16

 Note que eu fiz as seguintes hipóteses que o enunciado não deixa
 claras, mas que creio serem as mais razoáveis possíveis:
 i) A e B têm 50% de chance cada de vencer cada partida (que o próprio
 Fábio já havia destacado que não estava explícito no enunciado, e, na
 minha opinião, devia)
 ii) Partidas distintas são independentes entre si -- ou seja, que não
 interessa se A está para ser campeão ou B está desesperado, os times
 continuam com 50% de chance cada em cada partida, independentemente da
 história passada. Por isso que eu posso usar que p(1011) é o produto
 1/2.1/2.1/2.1/2.

 Então, p(A ser campeão)=p(11)+p(1011)=5/16. Tô com o Fábio (que,
 afinal, é da família Teixeira, então **não podia** estar enganado :)
 ).

 Abraço,
   Ralph


 2009/1/23 fabio henrique teixeira de souza fabiodja...@ig.com.br:
   Salhab, agradeço a ajuda. No entanto, acho que vc cometeu um engano.
 Não
  podemos calcular a chance de A ser campeão por 2 casos favoráveis em um
  total de 5, posto que os 5 não são equiprováveis. Estou errado?
  Abraço.
 
  2009/1/23 Marcelo Salhab Brogliato msbro...@gmail.com
 
  Olá Fábio,
  dado que A perdeu o primeiro jogo, temos as seguinte possiveis
  configuracoes para os demais jogos:
  vamos convencionar: 1 = A venceu, 0 = B venceu (ou A perdeu).
  0 (neste caso B é campeao)
  11 (neste caso A é campeao)
  100 (neste caso B é campeao)
  1010 (neste caso B é campeao)
  1011 (neste caso A é campeao)
  logo, temos 2/5 = 40% de chance de A ser campeão.
 
  Outra maneira seria calcular: P(A ser campeao | A perdeu a primeira
  partida) = P(A ser campeao e A perder a primeira partida)/P(A perder a
  primeira partida)
 
  P(A perder a primeira partida) = 1/2
  P(A ser campeao e A perder a primeira partida) = 2/10 [basta ver que
 temos
  2 casos favoraveis em 10, pois temos que considerar os casos em que A
 ganhou
  a primeira partida, que sao analogos aos acima]
 
  assim: P = (2/10)/(1/2) = 2/5 = 40%
 
  acho que é isso!!
  abraços,
  Salhab
 
 
 
 
  2009/1/22 fabio henrique teixeira de souza fabiodja...@ig.com.br
 
  Senhores, ao fazer a questão abaixo, encontrei como probabilidade 5/16.
  De saída, o problema não deixa claro que as probabilidades de vitória
 de A e
  de B são iguais. Considerei-as iguais (a 1/2). No entanto, o gabarito
  oficial diz que a declaração é CERTA. Gostaria que alguém também a
 fizesse
  para que eu possa ter idéia se estou errando ou acertando. Aguardo o
  retorno.
 
 
 
 
  Analise em certo ou errado:
 
  A etapa final de um torneio de futebol será disputada entre os
 
  times A e B, e o campeão será o time que vencer duas partidas
 
  seguidas ou um total de três partidas. Considerando que os jogos
 
  que terminarem empatados serão decididos nos pênaltis, de forma
 
  que sempre haja um vencedor, julgue os itens que se seguem.
 
  109
 
  A chance de o time A ser campeão tendo perdido o primeiro
 
  jogo é de 20%.
 
 

 =
 Instruções para entrar na lista, sair da lista e usar a lista em
 http://www.mat.puc-rio.br/~obmlistas/obm-l.html
 =



[obm-l] Dúvida de lógica

2009-01-23 Por tôpico fabio henrique teixeira de souza
Amigos, cá venho eu com mais um pepino.

Ontem, um amigo me perguntou: - 2 é primo e 3 é primo é uma proposição
simples ou composta?
Prontamente respondi que é composta.

A seguir, perguntou: - e a proposição 2 e 3 são números primos? É simples
ou composta.
Desta feita, não tive certeza, apenas a convicção de que é uma proposição
simples. No entanto, gostaria de sair da convicção para a certeza e, se
possível, descobrir alguma referência bibliográfica que embase esta certeza.

A importância dessa distinçaõ se dá por conta da negação.

Se, de fato, for simples, a negação de 2 e 3 são números primos será 2 e
3 não são números primos e não, 2 não é primo ou 3 não é primo. Parece-me
que a chave da questão é que o e que une 2 e 3 na expressão 2 e 3 são
números primos não é um conectivo lógico-matemático e sim uma conjunção da
língua portuguesa para construir um sujeito composto.

Aguardo as opiniões.

Um forte abraço.


Fabio Henrique TEIXEIRA de Souza (Olha aí, Ralph! Agora só vai em caixa
alta.) :)


[obm-l] Questão de Probabilidade - CESPE

2009-01-22 Por tôpico fabio henrique teixeira de souza
 Senhores, ao fazer a questão abaixo, encontrei como probabilidade 5/16. De
saída, o problema não deixa claro que as probabilidades de vitória de A e de
B são iguais. Considerei-as iguais (a 1/2). No entanto, o gabarito oficial
diz que a declaração é CERTA. Gostaria que alguém também a fizesse para que
eu possa ter idéia se estou errando ou acertando. Aguardo o retorno.




Analise em certo ou errado:

A etapa final de um torneio de futebol será disputada entre os

times A e B, e o campeão será o time que vencer duas partidas

seguidas ou um total de três partidas. Considerando que os jogos

que terminarem empatados serão decididos nos pênaltis, de forma

que sempre haja um vencedor, julgue os itens que se seguem.

109 A chance de o time A ser campeão tendo perdido o primeiro

jogo é de 20%.


Re: [obm-l] Questão CHATA ???

2008-12-18 Por tôpico Fabio Henrique
Rogerio , vc acertou a resposta é 30. Mas eu nao entendi o seu raciocinio.

2008/12/18 Bruno França dos Reis bfr...@gmail.com

 Boa Rogério, acabo de ver que cometi um erro na minha segunda solução (eu
 tinha afirmado que eram 32 formas, mas esqueci do detalhe de que cada
 soldado tinha que ter no mínimo um fuzil).

 --
 Bruno FRANÇA DOS REIS

 msn: brunoreis...@hotmail.com
 skype: brunoreis666
 tel: +33 (0)6 28 43 42 16

 http://www.brunoreis.com
 http://blog.brunoreis.com

 e^(pi*i)+1=0


 2008/12/18 Rogerio Ponce abrlw...@gmail.com

 Ola' pessoal,
 esse enunciado admite varias interpretacoes, pois os fuzis podem ser
 iguais ou diferentes entre si, e a palavra distribuicao pode se
 referir ao ato de distribuir (e nesse caso, se os fuzis forem
 diferentes entre si, devemos considerar a ordem em que eles sao
 entregues), ou ao resultado final do ato de distribuir (nesse caso,
 a ordem em que os fuzis foram entregues nao importa).

 Considerando fuzis diferentes , e apenas o resultado da entrega,
 teriamos a seguinte solucao, por exemplo:
 Cada fuzil tem 2 opcoes para ser entregue.
 Como sao 5 fuzis, ha' 2**5 = 32 opcoes.
 Como nao podemos ter todos os fuzis com o soldado A , ou todos com o
 soldado B, devemos eliminar 2 opcoes desse total.
 Assim, existem 30 formas de distribuicao dos fuzis.

 Observem que esta e' apenas uma das interpretacoes possiveis.
 []'s
 Rogerio Ponce




 2008/12/17 Fabio Henrique fabio.henrique.ara...@gmail.com:
  Essa questão é de um concurso que eu fiz e eu nao consigo entender o
  gabarito , espero que me ajudem , sem mais Fábio
 
  Dois soldados serão designados para uma mesma missão
  e  para  eles  serão  distribuídos  (sem  sobra)  5  fuzis  de  tal
  forma  que  cada  soldado  receba  ao menos  um  fuzil. Essa
  distribuição deverá ser feita de n formas.
  Então, pode-se afirmar que n vale : 
 
  Não vou postar a reposta pois pode interferir na resolução do problema.
  Desde já Obrigado.
 
  --
  Be Free
  Use LINUX
  Linux #244712
 

 =
 Instruções para entrar na lista, sair da lista e usar a lista em
 http://www.mat.puc-rio.br/~obmlistas/obm-l.htmlhttp://www.mat.puc-rio.br/%7Eobmlistas/obm-l.html
 =





-- 
Be Free
Use LINUX
Linux #244712


Re: [obm-l] Questão CHATA ???

2008-12-18 Por tôpico Fabio Henrique
eu pensei assim só que eu prensei em relação aos soldados. um arranjo de
distribuir 5 fuzis para 2 soldados
soldado 1 pode receber 5
soldado 2 pode receber 4

5*4=20 , mas esse raciocionio esta errado,. eu entendo pq o seu esta certo ,
mas não pq o meu está errado.

On Thu, Dec 18, 2008 at 1:57 PM, Carlos Nehab ne...@infolink.com.br wrote:

  Oi, gente,

 Eu acho que os fusíveis (como diz meu porteiro) são indistinguíveis e a
 questão é apenas saber os quantitativos diferentes de fusis que cada soldado
 diferente (distinguíveis) pode receber.  Não acham?

 Nehab

 PS: Oi, Ponce, agora sou um cara sério. Virei vovô de um lindo menino:
 Felipe. Saudades.

 Rogerio Ponce escreveu:

 Oi Fabio,
 conhece o problema sobre quantos pratos possiveis podem ser montados
 com uma salada, de um total de 5 saladas disponiveis, uma carne de um
 total de 3 carnes, um acompanhamento de um total de 5 acompanhamentos
 , e uma sobremesa de um total de 4 sobremesas?

 Voce sabe que existem 5*3*5*4=300 pratos possiveis, certo?

 Pois o problema dos fuzis e' a mesma coisa:
 Para o primeiro fuzil , existem 2 opcoes de entrega (soldado A ou soldado B)
 Para o segundo fuzil tambem existem 2 opcoes de entrega, e assim por diante.
 Ao final, podemos distribuir os 5 fuzis de 2*2*2*2*2 = 32 formas diferentes.

 Entretanto, como cada soldado recebe pelo menos 1 fuzil, devemos
 eliminar a distribuicao em que o soldado A nao recebeu fuzil algum, e
 a distribuicao em que o soldado B nao recebeu fuzil algum.
 O total sera' 32 - 2 = 30 fuzis.

 []'s
 Rogerio Ponce



 2008/12/18 Fabio Henrique fabio.henrique.ara...@gmail.com 
 fabio.henrique.ara...@gmail.com:


  Rogerio , vc acertou a resposta é 30. Mas eu nao entendi o seu raciocinio.

 2008/12/18 Bruno França dos Reis bfr...@gmail.com bfr...@gmail.com

  Boa Rogério, acabo de ver que cometi um erro na minha segunda solução (eu
 tinha afirmado que eram 32 formas, mas esqueci do detalhe de que cada
 soldado tinha que ter no mínimo um fuzil).

 --
 Bruno FRANÇA DOS REIS

 msn: brunoreis...@hotmail.com
 skype: brunoreis666
 tel: +33 (0)6 28 43 42 16
 http://www.brunoreis.comhttp://blog.brunoreis.com

 e^(pi*i)+1=0


 2008/12/18 Rogerio Ponce abrlw...@gmail.com abrlw...@gmail.com

  Ola' pessoal,
 esse enunciado admite varias interpretacoes, pois os fuzis podem ser
 iguais ou diferentes entre si, e a palavra distribuicao pode se
 referir ao ato de distribuir (e nesse caso, se os fuzis forem
 diferentes entre si, devemos considerar a ordem em que eles sao
 entregues), ou ao resultado final do ato de distribuir (nesse caso,
 a ordem em que os fuzis foram entregues nao importa).

 Considerando fuzis diferentes , e apenas o resultado da entrega,
 teriamos a seguinte solucao, por exemplo:
 Cada fuzil tem 2 opcoes para ser entregue.
 Como sao 5 fuzis, ha' 2**5 = 32 opcoes.
 Como nao podemos ter todos os fuzis com o soldado A , ou todos com o
 soldado B, devemos eliminar 2 opcoes desse total.
 Assim, existem 30 formas de distribuicao dos fuzis.

 Observem que esta e' apenas uma das interpretacoes possiveis.
 []'s
 Rogerio Ponce




 2008/12/17 Fabio Henrique fabio.henrique.ara...@gmail.com 
 fabio.henrique.ara...@gmail.com:


  Essa questão é de um concurso que eu fiz e eu nao consigo entender o
 gabarito , espero que me ajudem , sem mais Fábio

 Dois soldados serão designados para uma mesma missão
 e  para  eles  serão  distribuídos  (sem  sobra)  5  fuzis  de  tal
 forma  que  cada  soldado  receba  ao menos  um  fuzil. Essa
 distribuição deverá ser feita de n formas.
 Então, pode-se afirmar que n vale : 

 Não vou postar a reposta pois pode interferir na resolução do problema.
 Desde já Obrigado.

 --
 Be Free
 Use LINUX
 Linux #244712



  =
 Instruções para entrar na lista, sair da lista e usar a lista 
 emhttp://www.mat.puc-rio.br/~obmlistas/obm-l.html
 =


  --
 Be Free
 Use LINUX
 Linux #244712



  =
 Instruções para entrar na lista, sair da lista e usar a lista 
 emhttp://www.mat.puc-rio.br/~obmlistas/obm-l.html
 =



  =
 Instruções para entrar na lista, sair da lista e usar a lista em
 http://www.mat.puc-rio.br/~obmlistas/obm-l.html=




-- 
Be Free
Use LINUX
Linux #244712


[obm-l] Questão CHATA ???

2008-12-17 Por tôpico Fabio Henrique
Essa questão é de um concurso que eu fiz e eu nao consigo entender o
gabarito , espero que me ajudem , sem mais Fábio

Dois soldados serão designados para uma mesma missão
e  para  eles  serão  distribuídos  (sem  sobra)  5  fuzis  de  tal
forma  que  cada  soldado  receba  ao menos  um  fuzil. Essa
distribuição deverá ser feita de n formas.
Então, pode-se afirmar que n vale : 

Não vou postar a reposta pois pode interferir na resolução do problema.
Desde já Obrigado.

-- 
Be Free
Use LINUX
Linux #244712


Re: [obm-l] PSSC

2007-11-22 Por tôpico fabio henrique teixeira de souza
Obrigado.

Em 18/11/07, João Luís Gomes Guimarães [EMAIL PROTECTED] escreveu:

  Uma alternativa é procurar em sebos, você vai achar uma edição em
 português. Um ótimo site para procurar livros usados é um que congrega
 centenas de sebos por todo o país:

 www.estantevirtual.com.br

 Espero tê-lo ajudado. Um abraço,

 João Luís

 - Original Message -
 *From:* fabio henrique teixeira de souza [EMAIL PROTECTED]
 *To:* obm-l@mat.puc-rio.br
  *Sent:* Saturday, November 17, 2007 11:11 AM
 *Subject:* [obm-l] PSSC


 Alguém sabe onde posso comprar o PSSC, livro sensacional de física.

 Um abraço.

 Fabio




[obm-l] PSSC

2007-11-17 Por tôpico fabio henrique teixeira de souza
Alguém sabe onde posso comprar o PSSC, livro sensacional de física.

Um abraço.

Fabio


Re: [obm-l] OFF TOPIC absolutamente INCONVENIENTE

2007-10-02 Por tôpico fabio henrique teixeira de souza
Já trabalhei lá. Não achei que emitir minha opinião fosse causar tamanha
confusão. De qualquer forma, peço desculpas.

Em 27/09/07, Carlos Nehab [EMAIL PROTECTED] escreveu:

 Por favor, calma, Tio Cabri,

 Como eu me manifestei contra este tipo de off topic, gostaria de
 esclarecer minha posição.

 Veja se você concorda:

 Indicar livros na Lista para alguém que solicita dicas sobre um
 determinado tema, me parece perfeitamente adequado;  até dizer onde o livro
 pode ser encontrado, especialmente se é um clássico difícil de achar.   Não
 vejo nada de mais.

 Mas acho diferente, por exemplo, como já aconteceu aqui mais de uma vez,
 mandarem um email para a lista, do nada, dizendo que um determinado livro
 foi publicado e está à venda em algum lugar.  Você não concorda?  Haveria,
 subliminarmente, uma ação de marketing através da Lista, que não é adequado.

 Assim, penso que da mesma maneira, indicar Colégios, Cursos, etc, por
 melhor que seja sua causa (ajudar um amigo, como você disse) gera uma
 situação desconfortável na lista, pois muitos e muitos participantes já
 cursaram tais instituições, ou as cursam, ou até trabalham nas mesmas.
 Assim, mesmo uma pergunta aparentemente banal como a sua, gera uma situação
 quase de enquete e não é adequada.

 Veja, você mesmo foi testemunha, recentemente, de um email enviado por
 mim, infelizmente inadequado, por ter deixado margem a interpretações
 igualmente inadequadas.Como diz meu filho, sempre atento a NetEtiqueta
 (acredite, existe isto!)  pisei na bola.  E pedi desculpas.

 Uma maneira de resolver questões como a que você propôs seria talvez
 escrever em off para pessoas da lista com quem você mais se identifica e
 fazer sua enquete fora da Lista.  Eu já fiz isto dezenas de vezes e também
 já recebi dezenas de emails por fora...

 Finalizo apenas reafirmando que discordância não significa falta de
 respeito.   Significa apenas, entre pessoas de bem, que somos diferentes e
 pensamos diferente.

 Um grande abraço,
 Nehab



 Tio Cabri st escreveu:

 Tem alguém nessa lista que está de saca... e esse não sou 
 eu.itamentepertinente;
  O uso do off topic não pode ser direcionado a alguém como deseja o Sr
 Anselmo.
 O uso do off topic é algo fora da lista mas que muitos daqueles que usam a
 lista podem responder.
 E vou falar outra coisa:  vocês estão com o testosterona a mil, perguntei
 uma coisa importante para mim e sei que muitos dessa lista podem ajudar.
 Mas tem gente que gosta de criticar, acha bonito saca... os outros, vou
 fazer o quê?
 Quando eu morava no rio em 1970 poderia dizer que o curso impacto era um
 bom curso preparatório para ime ita.
 Hoje eu desconheço, e por isso perguntei na lista.
 Ora qual o problema disso?
 Muitos às vezes perguntam um bom livro e cada um dá a sua opinião e não me
 lembro de alguém sair em defesa dos autores desses livros citados.
 Fala sério, chega desse assunto, vamos voltar para o trabalho e para os
 estudos que é o fim dessa lista e àqueles que detestam o off topic
 simplesmente é só não ler.
 Abraços
 Cabri

 - Original Message -
 *From:* Anselmo Sousa [EMAIL PROTECTED]
 *To:* obm-l@mat.puc-rio.br
 *Sent:* Wednesday, September 26, 2007 10:26 PM
 *Subject:* RE: [obm-l] OFF TOPIC absolutamente INCONVENIENTE

 ACHO QUE ESSE NEGÓCIO DE OFF TOPIC CHEGOU A UM LIMITE...

 PODERÍAMOS PARAR, POR GENTILEZ, COM ESSA PRAGA...

 TODOS SABEMOS E-MAILS UNS DOS OUTROS...

 OFF TOPIC DEVE SER MESMO OFF TOPIC E, POR ISSO MESMO, NÃO DEVE APARECER
 AQUI...


 NÃO AO OF TOPIC!!!




  --
 Date: Wed, 26 Sep 2007 21:39:14 -0300
 From: [EMAIL PROTECTED]
 To: obm-l@mat.puc-rio.br
 Subject: [obm-l] OFF TOPIC absolutamente INCONVENIENTE

 Oi, gente,

 Com TODA franqueza, eu acho que por melhor que sejam as intenções de
 atender a algum amigo, este tipo de informação é mais do que OFF TOPIC:  é
 absolutamente INCONVENIENTE e sem qualquer cabimento.

 Há nesta lista, naturalmente,  inúmeros profissionais sérios que trabalham
 nestas instituições e, nestas condições, solicitações desta natreza sequer
 deveriam  ser formuladas.

 Nehab


 fabio henrique teixeira de souza escreveu:

 Ponto de Ensino

 Em 20/09/07, *Tio Cabri st* [EMAIL PROTECTED] escreveu:

 Bom dia, preciso indicar a um amigo o melhor curso preparatório para o
 concurso do IME - ITA na cidade do Rio de Janeiro.
 Gostaria da opinião dos senhores dessa lista.
 Obrigado
 Cabri

 =
 Instruções para entrar na lista, sair da lista e usar a lista em
 http://www.mat.puc-rio.br/~nicolau/olimp/obm-l.html
 =


 =
 Instruções para entrar na lista, sair da lista e usar a lista em
 http://www.mat.puc-rio.br/~nicolau/olimp/obm-l.html

Re: [obm-l] off topic: cursinho preparatorio

2007-09-26 Por tôpico fabio henrique teixeira de souza
Ponto de Ensino

Em 20/09/07, Tio Cabri st [EMAIL PROTECTED] escreveu:

 Bom dia, preciso indicar a um amigo o melhor curso preparatório para o
 concurso do IME - ITA na cidade do Rio de Janeiro.
 Gostaria da opinião dos senhores dessa lista.
 Obrigado
 Cabri

 =
 Instruções para entrar na lista, sair da lista e usar a lista em
 http://www.mat.puc-rio.br/~nicolau/olimp/obm-l.html
 =



[obm-l] Re:

2007-02-03 Por tôpico Fabio Henrique

EMULE , BITTORRENT

On 2/3/07, André Smaira [EMAIL PROTECTED] wrote:


Vcs poderiam me informar onde acho o Maple mais recente possivel e de
graça para baixar?
Eu tinha o original mas fiz uma modificações no meu pc e nao consegui
instalar +!!!

__
Fale com seus amigos de graça com o novo Yahoo! Messenger
http://br.messenger.yahoo.com/





--
Be Free
Use LINUX
Linux #244712


Re: [obm-l] Motivos para votar em LULA!!!

2006-10-23 Por tôpico Fabio Henrique
Primeiro sou Eleitor do Lula Segundo voce está enganado meu caro. Caso o presidente eleito sofra um processo de IMpeachmente antes dos dois anos de mandato , será feita nova eleição. Porém IMpeachemnt é um processo politico e com 62% da populção apoiando o Presidente , acho díficil , esperem mais 4 anos e tente a sorte com o Aecio. Lula de Novo , com a força do POvo.

On 10/22/06, Vitor Tomita Silva [EMAIL PROTECTED] wrote:
Olhem agora o nível dos eleitores do Alckmin, generalizando demais.Claro, o que o sr. eleitor do Lula fez foi bem pior... ainda mais mandando
propaganda numa lista de matemática, sendo que a imensa maioria da galera deexatas odeia o Lula. Hugo, falta-lhe raciocínio lógico-matemático.Acho que o que falta nessa eleição não é só candidato, é eleitor bonzinho
também. Se fôssemos todos fofos e discutíssemos MATEMÁTICA nessa lista,seria bem melhor. Afinal, todos sabemos que, com a OAB já dizendo quederruba o Lula se ele for reeleito, o Alckmin (provavelmente) assume até se
perder.From: Fernando A Candeias [EMAIL PROTECTED]Reply-To: obm-l@mat.puc-rio.brTo: 
obm-l@mat.puc-rio.brSubject: Re: [obm-l] Motivos para votar em LULA!!!Date: Sun, 22 Oct 2006 11:02:25 -0200*Acho que é uma atitude que reflete bem o nível dos eleitores do Lula: vale
tudo* Em 22/10/06, Hugo Leonardo da Silva Belisário[EMAIL PROTECTED]escreveu:Me limito a citar vário links nos quais fundamento meu voto em LULA para
presidente. Leiam,http://carosamigos.terra.com.br/da_revista/edicoes/ed114/valeapena.asp
http://forums.ecomm.com.br/cgi/dnewsweb.exe?cmd=articlegroup=forum.carosamigositem=7215utag=
http://forums.ecomm.com.br/cgi/dnewsweb.exe?cmd=articlegroup=forum.carosamigositem=7241utag=
http://forums.ecomm.com.br/cgi/dnewsweb.exe?cmd=articlegroup=forum.carosamigositem=7336utag=
http://forums.ecomm.com.br/cgi/dnewsweb.exe?cmd=articlegroup=forum.carosamigositem=7428utag=
http://www.adital.com.br/site/noticia.asp?lang=PTcod=24972
http://www.adital.com.br/site/noticia.asp?lang=PTcod=24993http://www.adital.com.br/site/noticia.asp?lang=PTcod=24973
http://www.adital.com.br/site/noticia.asp?lang=PTcod=24969
http://www.adital.com.br/site/noticia.asp?lang=PTcod=24964http://www.adital.com.br/site/noticia.asp?lang=PTcod=24963
http://www.adital.com.br/site/noticia.asp?lang=PTcod=24962
http://www.adital.com.br/site/noticia.asp?lang=PTcod=22858O que acham?___
O Yahoo! está de cara nova. Venha conferir!http://br.yahoo.com=
Instruções para entrar na lista, sair da lista e usar a lista emhttp://www.mat.puc-rio.br/~nicolau/olimp/obm-l.html=
--Fernando A Candeias_O Windows Live Spaces é seu espaço na internet com fotos (500 por mês), blog
e agora com rede social http://spaces.live.com/=Instruções para entrar na lista, sair da lista e usar a lista em
http://www.mat.puc-rio.br/~nicolau/olimp/obm-l.html=
-- Be Free Use LINUX 


Re: [obm-l] Comunicado

2006-09-14 Por tôpico Fabio Henrique
É triste , porém a puniçao foi exemlar como deve ser.On 9/14/06, Artur Costa Steiner [EMAIL PROTECTED]
 wrote:Este fato eh ainda mais triste e lamentavel por envolver um estudante de
nivel medio, muito provavelmente um adolescente, alguem numa idade em quecostuma haver idealismo. Reflexo, talvez, do que acontece em nosso pais.Esperemos que este triste episodio ao menos sirva para que este estudante
reflita e muda suas atitudes para o futuro.Artur-Mensagem original-De: [EMAIL PROTECTED] [mailto:
[EMAIL PROTECTED]]Emnome de Olimpiada Brasileira de MatematicaEnviada em: quinta-feira, 14 de setembro de 2006 02:47Para: Lista de discussaoAssunto: [obm-l] Comunicado*
COMUNICADO*Rio de Janeiro, 13 de Setembro de 2006À Comunidade OlímpicaOlimpíada Brasileira de Matemática - OBMPrezados alunos, professores e comunidade olímpica:
No dia 3 de Setembro de 2006, a Olimpíada Brasileira de Matemáticarecebeu, via e-mail, numerosas denúncias segundo as quais um estudantede Ensino Médio, participante do Nível 3 na XXVIII Olimpíada Brasileira
de Matemática – OBM, tentou de forma ilícita conseguir respostas paraduas questões da prova da OBM, que foi aplicada em todo o territórionacional dia 2/09/2006. Após uma rigorosa investigação e com base nas
provas apresentadas, ficou demonstrado que o aluno, devidamenteidentificado, teve acesso às duas questões da prova diretamente nocolégio onde estuda e que tentou conseguir as soluções postando oconteúdo para uma comunidade existente em um conhecido site de
relacionamentos na Internet.Diante desta grave quebra de sigilo por parte da instituição, e da clarae total violação a todos os princípios de honra olímpica por parte doaluno, a Comissão Nacional de Olimpíadas de Matemática determinou a
imediata desclassificação do aluno, bem como também a desclassificaçãoda instituição de ensino na XXVIII Olimpíada Brasileira de Matemática de2006. Ficou também determinada a proibição, tanto para o aluno quanto
para a instituição, de participar da competição durante o ano de 2007 emtodos os seus níveis de participação. A medida adotada pela ComissãoNacional de Olimpíadas de Matemática é irrevogável, não cabendo recursos
por parte dos envolvidos.Espera-se que atitudes como as anteriormente mencionadas não voltem aocorrer dentro da comunidade olímpica, e ressaltamos que a OlimpíadaBrasileira de Matemática é uma atividade de livre participação, tanto
para os alunos quanto para os professores, e que tem como finalidadeprincipal estimular o estudo da Matemática entre os jovens, aperfeiçoarprofessores e propiciar uma melhoria do ensino e do aprendizado desta
matéria nas escolas brasileiras.Cordialmente,Comissão Nacional de Olimpíadas de MatemáticaOlimpíada Brasileira de Matemática - OBM=
Instruções para entrar na lista, sair da lista e usar a lista emhttp://www.mat.puc-rio.br/~nicolau/olimp/obm-l.html=
=Instruções para entrar na lista, sair da lista e usar a lista emhttp://www.mat.puc-rio.br/~nicolau/olimp/obm-l.html
=-- Be Free Use LINUX


Re: [obm-l] Fabio Henrique equivocado

2004-09-27 Por tôpico Fabio Henrique
 Desculpem. Foi por puro zelo pela nossa lista. Afinal, já aconteceu antes. 
Foi um comentário do tipo li, entendi errado e não gostei. 



Em 27 Sep 2004, [EMAIL PROTECTED] escreveu: 


O objetivo desta lista é disseminar matemática de todas as formas sempre 
com 
respeito em todos os aspectos. Creio que o O Fábio se equivocou devido a a 
contecimentos recentes. O que eu mandei são livros disponibilizados pelos 
seus autores. E como todos nós precisamos ler muito para continuarmos 
ajudando uns aos outros, essa foi minha intenção e creio que é a intenção 
dos autores. O própio Professor Nicolau vez ou outra oferece seu livro 
nesta 
lista e onde está erro? 
Creio que o comentário foi algo do tipo  não li e não gostei 
(^_^) 
 
_ 
MSN Messenger: converse com os seus amigos online. 
http://messenger.msn.com.br 
 
Instruções 
para entrar na lista, sair da lista e usar a lista em 
http://www.mat.puc-rio.br/~nicolau/olimp/obm-l.html 
 
 
-- 

_
Quer mais velocidade?
Só com o acesso Aditivado iG, a velocidade que você quer na hora que você precisa.
Clique aqui: http://www.acessoaditivado.ig.com.br



Re: [obm-l] Dentro- da- Lei... Livros Gratuitos

2004-09-26 Por tôpico Fabio Henrique
 Será que você poderia divulgar este tipo de material em particular, entre 
os seus. Ou quem sabe, inaugurar uma página para este tipo de prática. 
Talvez você não tenha se dado conta dos problemas que esta lista pública 
pode ter por causa de procedimentos como este. 
Grato. 
Fabio Henrique 


Em 26 Sep 2004, [EMAIL PROTECTED] escreveu: 


Não é preciso ser fora-da-lei para se conseguir muitos livros, apostilas e 
monografias gratuitas de Matemática na internet. Aí vão alguns endereços 
e... divirtam-se 
 
http://www.elprisma.com/apuntes/apuntes.asp?categoriap4 
 
http://www.numbertheory.org/ntw/lecture_notes.html 
 
http://www.math.miami.edu/~ec/book/ 
 
http://www.maths.nott.ac.uk/personal/jec/courses/G13NUM/#link 
 
http://f2.org/links/books.html 
 
Tem mais outros, vou procurar com mais paciência e qualquer dia coloco na 
lista 
 
(^_^) 
 
_ 
MSN Messenger: converse com os seus amigos online. 
http://messenger.msn.com.br 
 
Instruções 
para entrar na lista, sair da lista e usar a lista em 
http://www.mat.puc-rio.br/~nicolau/olimp/obm-l.html 
 
 
-- 

_
Quer mais velocidade?
Só com o acesso Aditivado iG, a velocidade que você quer na hora que você precisa.
Clique aqui: http://www.acessoaditivado.ig.com.br



Re: RES: [obm-l] [PELEJA] Desafio do Google

2004-09-18 Por tôpico Fabio Henrique
 Onde posso obter o mathematica? 


Em 18 Sep 2004, [EMAIL PROTECTED] escreveu: 


Bom, no meu caso eu reconheci os primeiros números como as casas 
iniciais de e e depois usei o Mathematica para verificar de onde saiam 
as outras. 
Daí, foi só perceber a lógica envolvida e procurar a 720ª casa 
decimal... 
 
Um abraço, 
 
Guilherme. 
 
-Mensagem original- 
De: [EMAIL PROTECTED] [mailto:[EMAIL PROTECTED] Em 
nome de David M. Cardoso 
Enviada em: sábado, 18 de setembro de 2004 01:00 
Para: [EMAIL PROTECTED] 
Assunto: Re: [obm-l] [PELEJA] Desafio do Google 
 
Como faz pra achar isso? 
Apelação/Força bruta computacional? 
E se for um problema te testar exaustivamente vários números (problema 
computacional), como fazer pra encontrar a n-ésima casa decimal da 
constante de euler? 
 
[]'s 
David 
 
 
= 
Instruções para entrar na lista, sair da lista e usar a lista em 
http://www.mat.puc-rio.br/~nicolau/olimp/obm-l.html 
 
= 
 
= 
Instruções para entrar na lista, sair da lista e usar a lista em 
http://www.mat.puc-rio.br/~nicolau/olimp/obm-l.html 
= 
 
-- 

_
Quer mais velocidade?
Só com o acesso Aditivado iG, a velocidade que você quer na hora que você precisa.
Clique aqui: http://www.acessoaditivado.ig.com.br



Re: [obm-l] Re: IME

2004-07-23 Por tôpico Fabio Henrique
Wallace, é exatamente este endereço que tenho. No entanto, não consigo 
entrar na página de jeito nenhum. 
Veja se consegue e me dê retorno. 
Obrigado. 


Em 21 Jul 2004, [EMAIL PROTECTED] escreveu: 


Ola Fabio, 
 
 A pagina do Prof. Ph.D Sergio Lima Netto é a seguinte: 
http://www.lps.ufrj.br/~sergioln/ 
 
Um abraço! 
 
Wallace Alves Martins 
Laboratorio de Processamento de Sinais/UFRJ 
 
Fabio Henrique escreve: 
 
 Alguém poderia me escrever o endereço da página do professor Sérgio que 
 contém as provas do Ime? 
 Grato. 
 
 
_ 
 Quer mais velocidade? 
 Só com o acesso Aditivado iG, a velocidade que você quer na hora que você 
precisa. 
 Clique aqui: http://www.acessoaditivado.ig.com.br 
 
 
= 
Instruções para entrar na lista, sair da lista e usar a lista em 
http://www.mat.puc-rio.br/~nicolau/olimp/obm-l.html 
= 
 
-- 

_
Quer mais velocidade?
Só com o acesso Aditivado iG, a velocidade que você quer na hora que você precisa.
Clique aqui: http://www.acessoaditivado.ig.com.br



Re: [obm-l] Re: IME

2004-07-23 Por tôpico Fabio Henrique
Se não for incômodo, agradeceria muito. 


Em 23 Jul 2004, [EMAIL PROTECTED] escreveu: 


Fábio. 
 
 Eu consegui entrar e baixar o arquivo. 
 Zipado, o tamanho é de cerca de 650 kb. 
 Se você quiser, posso enviar para você. 
 
 Abraços. 
 Hugo. 
 
 Fabio Henrique wrote: 
 
Wallace, é exatamente este endereço que tenho. No entanto, não consigo 
entrar na página de jeito nenhum. 
Veja se consegue e me dê retorno. 
Obrigado. 
 
-- 

_
Quer mais velocidade?
Só com o acesso Aditivado iG, a velocidade que você quer na hora que você precisa.
Clique aqui: http://www.acessoaditivado.ig.com.br



[obm-l] IME

2004-07-21 Por tôpico Fabio Henrique
Alguém poderia me escrever o endereço da página do professor Sérgio que 
contém as provas do Ime? 
Grato. 

_
Quer mais velocidade?
Só com o acesso Aditivado iG, a velocidade que você quer na hora que você precisa.
Clique aqui: http://www.acessoaditivado.ig.com.br



[obm-l] Re: ção do teorema de Fermat

2004-06-11 Por tôpico Fabio Henrique
Pode mandar para mim? 
Grato. 


Em 10 Jun 2004, [EMAIL PROTECTED] escreveu: 

Como faço para disponibilizar o arquivo para o grupo?? 
Formato: pdf 
Tamanho: 864 Kb 
Abraço. 
 
= 
Instruções para entrar na lista, sair da lista e usar a lista em 
http://www.mat.puc-rio.br/~nicolau/olimp/obm-l.html 
= 
 
-- 

_
Quer mais velocidade?
Só com o acesso Aditivado iG, a velocidade que você quer na hora que você precisa.
Clique aqui: http://www.acessoaditivado.ig.com.br



Re: [obm-l] Teo. de Wilson

2004-06-06 Por tôpico Fabio Henrique
Vou começar com um exemplo numérico. 

Seja p=11 

(p-1)! = 10.9.8.7.6.5.4.3.2.1 

Observe que 9.5 = 1 (mod 11) ENTENDA O SINAL DE = COMO CONGRUENTE. 
8.7 = 1 (mod 11) 
6.2 = 1 (mod 11) 
3.4 = 1 (mod 11) 
Assim, (p-1)! = 10.1.1.1.1.1 = -1.1.1.1.1.1 = -1 

Para p primo qualquer, sabemos que todos os elementos de {1,2,3,4,...,p-1} 
têm inverso multiplicativo. Além disso, o inverso de 1 e 1 (mod p) e o 
inverso de p-1 é p-1 (mod p). Assim, 2.3.4. ... .p-2 = 1 (mod p) 

(p-1)! = 1.2.3. ... .p-2.p-1 = 1.1.1. ... .1.p-1 (mod p) 
(p-1)! = 1.1.1. ... -1 (mod p) 

Em  6 Jun 2004, [EMAIL PROTECTED] escreveu: 

Pessoal, como provo o teo. de wilson,ou seja, 
se p é primo entao (p-1)!+1 é congruente a 0 módulo p 
 
Atenciosamente, 
 
Engenharia Elétrica - UNESP Ilha Solteira 
Osvaldo Mello Sponquiado 
Usuário de GNU/Linux 
 
__ 
Acabe com aquelas janelinhas que pulam na sua tela. 
AntiPop-up UOL - É grátis! 
http://antipopup.uol.com.br/ 
 
= 
Instruções para entrar na lista, sair da lista e usar a lista em 
http://www.mat.puc-rio.br/~nicolau/olimp/obm-l.html 
= 
 
-- 

_
Quer mais velocidade?
Só com o acesso Aditivado iG, a velocidade que você quer na hora que você precisa.
Clique aqui: http://www.acessoaditivado.ig.com.br



Re: [obm-l] iso-8859-1?b?UmU6W29ibS1sXSB SZTogW29ibS1sXSBSZTpbb2JtLWx dIFJlOiBbb2Jt iso-8859-1?b?LWxdIFJlOiBbb2I gbS1sXSBSZTpbb2JtLWxdIFJlOiB bb2IgbS1sXSBS iso-8859-1?b?ZTpbb2JtLWwgXSB Db2zpZ2lvIE5hdmFs

2004-06-04 Por tôpico Fabio Henrique
Se não for incômodo... 
Obrigado. 



Em  3 Jun 2004, [EMAIL PROTECTED] escreveu: 

Que nd, com maior prazer... ja anotei o mail de vcs... 
matematica sempre convem! 
falow 
 
[Ao som de Numb - Linkin Park] 
 
 Oi Osvaldo se não for exploração , quando mandar 
para os colegas no final da 
 semana ,gostaria muito que mandaçe pra mim tb, 
dedsde ja agradeço.Meu 1/2 é 
 [EMAIL PROTECTED] 
 - Original Message - 
 From: Osvaldo [EMAIL PROTECTED] 
 To: obm-l 
 Sent: Wednesday, June 02, 2004 8:52 PM 
 Subject: [obm-l] Re:[obm-l] Re: [obm-l] Re: [obm-l] 
Re:[obm-l] Re: [ob m-l] 
 Re:[obm-l] Colégio Naval 
 
 
  ta legal... mando sim.. so no fim d semana! 
  falow 
  
  
  
   Se não for muito incomodo, poderia também mandar 
para 
  este e-mail 
   [EMAIL PROTECTED] 
   
   Agradeço desde de já. 
   
   - Original Message - 
   From: João Luís 
   To: [EMAIL PROTECTED] 
   Sent: Wednesday, June 02, 2004 9:50 AM 
   Subject: [obm-l] Re: [obm-l] Re:[obm-l] Re: 
[obm-l] 
  Re:[obm-l] Colégio Naval 
   
   
   Ok, muito obrigado. Qualquer coisa, pode ir 
  mandando aos poucos, se não for te dar muito 
trabalho. 
  Mesmo porque, aqui não é banda larga não, hehehe.. 
   - Original Message - 
   From: Osvaldo 
   To: obm-l 
   Sent: Tuesday, June 01, 2004 10:29 PM 
   Subject: [obm-l] Re:[obm-l] Re: [obm-l] Re: 
[obm- 
  l] Colégio Naval 
   
   
   E ai Joao! 
   Vo tentar mandar ao seu e mail no fim de 
semana, 
  pq eu 
   levo o pc no meu primo... e la é banda larga. 
  Falou! 
   
   
   
Olá pessoal. 
Aproveitando a deixa: como sou professor de 
   Matemática (em Belo Horizonte), interesso-me 
e 
  muito 
   por provas de vestibulares e materail afim. A 
  todos 
   vocês que têm essas coleções, como o 
Osvaldo, 
  ou que 
   possam me fornecer qualquer fonte de onde 
  encontrar 
   esse material, eu agradeceria muito que me 
  enviasse. 
   Não sei se essa lista é exatamente o espaço 
para 
  isso, 
   então podem mandar em PVT para João Luís, 
   [EMAIL PROTECTED] 
Agradeço antecipadamente a todos que 
  colaborarem!!! 
- Original Message - 
From: Osvaldo 
To: obm-l 
Sent: Sunday, May 30, 2004 4:35 AM 
Subject: [obm-l] Re:[obm-l] Colégio Naval 


Boa Fábio, acessei seu site, me rendeu 
mais 
  algumas 
provas para minha coleçao em meu 
winchester. 
  falow 
   ai! 



 Disponibilizei 13 anos de provas do 
colégio 
  Naval. 
 Quando tiver tempo, coloco mais. 
 
 Espero que seja útil. 
 
 
   
  http://construtor.aprendebrasil.com.br/fabio1766469 
 
 

Atenciosamente, 

Engenharia Elétrica - UNESP Ilha Solteira 
Osvaldo Mello Sponquiado 
Usuário de GNU/Linux 




   
  
___ 
_ 
   __ 
Acabe com aquelas janelinhas que pulam 
na sua 
  tela. 
AntiPop-up UOL - É grátis! 
http://antipopup.uol.com.br/ 




   
  
=== 
= 
   = 
Instruções para entrar na lista, sair da 
  lista e 
   usar a lista em 
http://www.mat.puc- 
rio.br/~nicolau/olimp/obm- 
  l.html 

   
  
=== 
= 
   = 

   
   Atenciosamente, 
   
   Engenharia Elétrica - UNESP Ilha Solteira 
   Osvaldo Mello Sponquiado 
   Usuário de GNU/Linux 
   
   
   
   
  
___ 
_ 
  __ 
   Acabe com aquelas janelinhas que pulam na sua 
  tela. 
   AntiPop-up UOL - É grátis! 
   http://antipopup.uol.com.br/ 
   
   
   
   
  
=== 
= 
  = 
   Instruções para entrar na lista, sair da 
lista e 
  usar a lista em 
   http://www.mat.puc-rio.br/~nicolau/olimp/obm- 
  l.html 
   
  
=== 
= 
  = 
   
  
  Atenciosamente, 
  
  Engenharia Elétrica - UNESP Ilha Solteira 
  Osvaldo Mello Sponquiado 
  Usuário de GNU/Linux 
  
  
  
  
___ 
___ 
  Acabe com aquelas janelinhas que pulam na sua tela. 
  AntiPop-up UOL - É grátis! 
  http://antipopup.uol.com.br/ 
  
  
  
  
=== 
== 
  Instruções para entrar na lista, sair da lista e 
usar a lista em 
  http://www.mat.puc-rio.br/~nicolau/olimp/obm-l.html 
  
=== 
== 
  
 
 
 
=== 
== 
 Instruções para entrar na lista, sair da lista e 
usar a lista em 
 http://www.mat.puc-rio.br/~nicolau/olimp/obm-l.html 
 
=== 
== 
 
 
Atenciosamente, 
 
Engenharia Elétrica - UNESP Ilha Solteira 
Osvaldo Mello 

Re:[obm-l] elipse

2004-06-01 Por tôpico Fabio Henrique
Vocês não receberam a solução sem uso de derivadas? 



Em 1 Jun 2004, [EMAIL PROTECTED] escreveu: 

É claro que não está certo, até porque as equações encontradas não 
representam retas. 
 
 == 
 Mensagem  enviada  pelo  CIP  WebMAIL  - Nova Geração - v. 2.1 
 CentroIn Internet Provider           http://www.centroin.com.br 
 Tel: (21) 2542-4849, (21) 2295-3331        Fax: (21) 2295-2978 
 Empresa 100% Brasileira - Desde 1992 prestando servicos online 
 
 -- Original Message --- 
 From: Jefferson Franca 
 To: [EMAIL PROTECTED] 
 Sent: Mon, 31 May 2004 22:21:51 -0300 (ART) 
 Subject: Re:[obm-l] elipse 
 
  Valeu! Uma curiosidade: E sem derivada? Como ficaria? 
  
  Osvaldo [EMAIL PROTECTED] wrote: Posso decompor esta eq. ai em duas 
funçoes 
  f(x)_1 = +sqrt(1-(x/2)^2) 
  f(x)_2 = -sqrt(1-(x/2)^2) 
  (x_0,y_o)=(3,2) 
  Uma saída é utilizar y-y_0=y'.(x-x_0) (y'=d(f(x))/dx) 
  como reta tangente em (x_0,y_0) 
  
  Da primeira funçao vem que y-2=-x(x-3)/sqrt(1-(x/2)^2) 
  Da segunda funçao vem que y-2=x(x-3)/sqrt(1-(x/2)^2) 
  
  Bom, não sei se ta certo, se estiver a eq. vai 
  corresponder a 1-(x/2)^2=x(x-3)/(y-2) 
  
  falow ai 
  
   Será q alguém poderia me ajudar com a questão: 
  Determine a equação das tangentes à elipse (x^2)/4 + 
  (y^2) = 1, que passam pelo ponto P(3,2). 
   
   
   4-x^2 /4 -2x 
  
  - 
   
   - 
   Yahoo! Mail - Participe da pesquisa global sobre o 
  Yahoo! Mail. Clique aqui! 
  
  Atenciosamente, 
  
  Engenharia Elétrica - UNESP Ilha Solteira 
  Osvaldo Mello S! ponquiado 
  Usuário de GNU/Linux 
  
  
__ 
  Acabe com aquelas janelinhas que pulam na sua tela. 
  AntiPop-up UOL - É grátis! 
  http://antipopup.uol.com.br/ 
  
  
= 
  Instruções para entrar na lista, sair da lista e usar a lista em 
  http://www.mat.puc-rio.br/~nicolau/olimp/obm-l.html 
  
= 
  
  
 Yahoo! Mail - Participe da pesquisa global sobre o Yahoo! Mail. Clique 
aqui! 
 --- End of Original Message --- 
 
-- 

_
Quer mais velocidade?
Só com o acesso Aditivado iG, a velocidade que você quer na hora que você precisa.
Clique aqui: http://www.acessoaditivado.ig.com.br



Re: [obm-l] Problema_de_combinatória

2004-05-31 Por tôpico Fabio Henrique
Na verdade você quer saber quantos números são divisíveis por 6 entre 100 e 
999. Nesta faixa, o 1º múltiplo de 6 é 102=6x17 e o último é 996=6x166. 
Agora conte quantos números você tem de 17 a 166. Resp: 166-17+1=150 
Forte abraço. 
Fabio Henrique. 


Em 30 May 2004, [EMAIL PROTECTED] escreveu: 

Olá pessoal, é um prazer participar desta 
lista. 
 
 Resolvi o problema abaixo dividindo-o em muitos 
casos. 
 
 Quantos números de 3 algarismos distintos são 
divisíveis por 6? 
 
 Peço sugestões para uma resolução mais 
suscinta. 
 
 Agradeço 
 
-- 

_
Quer mais velocidade?
Só com o acesso Aditivado iG, a velocidade que você quer na hora que você precisa.
Clique aqui: http://www.acessoaditivado.ig.com.br



Re: [obm-l] CONSELHOS PEDAGÓGICOS [off-topic]

2004-05-29 Por tôpico Fabio Henrique
Sinceramente, acho que você deveria ensinar aos alunos a tal da divisão. É 
mais importante que trigonô. 


Em 28 May 2004, [EMAIL PROTECTED] escreveu: 

Olá amigos da lista, 
 
estou diante de uma situação-problema e gostaria muito de receber uma 
sugestão. 
 
Estou fazendo estágio numa escola onde funciona o EJA, educação para jovens 
e adultos. Posso dizer que o ensino lá é precário, os alunos estão vendo 
trigonometria sem saber dividisão!!! Não sabem tabuada Nada, nada, nada. 
 
Fiquei extremamente triste com a situação e quero tentar mudá-la. Não sei 
como devo proceder, pois eles não têm base alguma... 
 
Por onde devo começar? Tabuada, soma??? (...) 
 
Estou perdido, completamente perdido... 
 
Obrigado, 
 
[]'s 
 
-- 

_
Quer mais velocidade?
Só com o acesso Aditivado iG, a velocidade que você quer na hora que você precisa.
Clique aqui: http://www.acessoaditivado.ig.com.br



Re: [obm-l] Re:_CONSELHOS_PEDAGÓGICOS_[o ff-topic]

2004-05-29 Por tôpico Fabio Henrique
Eric, poderia enviar para mim também? 
Obrigado. 
FH 


Em 29 May 2004, [EMAIL PROTECTED] escreveu: 

 o ensino lá é precário, os 
 alunos estão vendo 
 trigonometria sem saber dividisão!!! Não sabem 
 tabuada Nada, nada, nada. 
 
Tenho um material estilo metodo Kumon que 
talvez pudesse ser usado, caso os alunos (ou a escola) 
estivessem dispostos a pagarem as xerox. Estou 
comecando a usar este sistema com um sobrinho meu. 
Posso te mandar alguma coisa nos formatos .xls ou .pdf 
(ou .doc se tiver Word) 
te mandar alguma 
 
[ ]'s 
 
Eric. 
 
 Fiquei extremamente triste com a situação e quero 
 tentar mudá-la. Não sei 
  como devo proceder, pois eles não têm base alguma... 
 Por onde devo começar? Tabuada, soma??? (...) 
 Estou perdido, completamente perdido... 
 Obrigado, 
 
= 
Instruções para entrar na lista, sair da lista e usar a lista em 
http://www.mat.puc-rio.br/~nicolau/olimp/obm-l.html 
= 
 
-- 

_
Quer mais velocidade?
Só com o acesso Aditivado iG, a velocidade que você quer na hora que você precisa.
Clique aqui: http://www.acessoaditivado.ig.com.br



Re: [obm-l] elipse

2004-05-29 Por tôpico Fabio Henrique
Sua reta Y=Ax+B passa pelo ponto (3,2). Assim, B = 2-3A. 

Resolva o sistema formado pela equação da elipse e pela equação da reta (não 
esqueça que B=2-3A). Você cairá em uma equação do 2º grau. Para que a reta 
seja tangente, delta deve ser nulo. Se delta negativo, a reta é exterior sem 
interseção alguma e, quando delta positivo, a reta atravessa a elipse. 
Eu achei y = [(24+sqrt21).x + (2-3sqrt21)]/37 
e y = [(24-sqrt21).x + (2+3sqrt21)]/37 




Em 29 May 2004, [EMAIL PROTECTED] escreveu: 

Será q alguém poderia me ajudar com a questão: Determine a equação das 
tangentes à elipse (x^2)/4 + (y^2) = 1, que passam pelo ponto P(3,2). 
 
-- 

_
Quer mais velocidade?
Só com o acesso Aditivado iG, a velocidade que você quer na hora que você precisa.
Clique aqui: http://www.acessoaditivado.ig.com.br



Re: [obm-l] iso-8859-1?b?UmU6W29ibS1sXSB SZTogW29ibS1sXSBSZTogW29ibS1 sXSBPUyBO2k1F iso-8859-1?b?Uk9TIERPIEFDQVN PIQ==

2004-05-26 Por tôpico Fabio Henrique
Há uma outra interpretação equivocada: 
temos 6 resultados pares (0,2,4,6,8,10) contra 5 ímpares (1,3,5,7,9) 





Em 26 May 2004, [EMAIL PROTECTED] escreveu: 

Teve uma questão da OBM, acho que foi do ano passado, 
na prim. fase em que dois jogadores usavam 1,2,... ou 5 
dedos, e dava 64 por cento de chance de quem escolheu 
par vencer. 
Afinal, se cada um usa de 0 a 10 dedos, quem vence? 
 
 On Tue, May 25, 2004 at 11:18:00PM -0300, Fellipe 
Rossi wrote: 
  PASMEM! O jogo do par ou ímpar é, sem sombra de 
dúvidas, favorável a C. 
  Abraços! 
  
  Por quê? 
  
  Rossi 
 
 Talvez outros adivinhem melhor do que eu, mas eu não 
tenho a menor idéia 
 de quem o o que seja este C. que estaria sendo 
favorecido. 
 
 Meu melhor palpite é que estamos falando daquele 
raciocínio 
 (completamente errado) que diz que PAR tem 
probabilidade maior 
 do que ÍMPAR de ganhar, pois para PAR há duas 
possibilidades 
 (os dois jogadores jogam números pares; os dois 
jogadores 
 jogam números ímpares) enquenato para ÍMPAR só há uma 
possibilidade 
 (os dois jogadores escolhem números de paridades 
diferentes). 
 
 []s, N. 
 
 
= 
 Instruções para entrar na lista, sair da lista e usar 
a lista em 
 http://www.mat.puc-rio.br/~nicolau/olimp/obm-l.html 
 
 
= 
 
 
Atenciosamente, 
 
Engenharia Elétrica - UNESP Ilha Solteira 
Osvaldo Mello Sponquiado 
Usuário de GNU/Linux 
 
__ 
Acabe com aquelas janelinhas que pulam na sua tela. 
AntiPop-up UOL - É grátis! 
http://antipopup.uol.com.br/ 
 
= 
Instruções para entrar na lista, sair da lista e usar a lista em 
http://www.mat.puc-rio.br/~nicolau/olimp/obm-l.html 
= 
 
-- 

_
Quer mais velocidade?
Só com o acesso Aditivado iG, a velocidade que você quer na hora que você precisa.
Clique aqui: http://www.acessoaditivado.ig.com.br



[obm-l] Re: dúvida

2004-05-12 Por tôpico Fabio Henrique
Achei 365. 

(x+y)(x-y)=27 
x+y=27 e x-y=1 = x=14 e y=13 = x^2+y^2=365 
x+y=9 e x-y=3 = x=6 e y=3 
Em 12 May 2004, [EMAIL PROTECTED] escreveu: 

a diferença entre o quadrado de dois números 
naturais é 27. uma ´possível soma dos quadrados desses números : 
 
 a)529 
 b)625 
 c)729 
 d)841 
 
-- 

_
Voce quer um iGMail protegido contra vírus e spams? 
Clique aqui: http://www.igmailseguro.ig.com.br



Re: [obm-l] Problema da raiz

2004-05-12 Por tôpico Fabio Henrique
Como assim, 16? Não concordo. Desta forma, a minha idade fica raiz quarta 
de 30. 


m 12 May 2004, [EMAIL PROTECTED] escreveu: 

Junior said: 
 Problema: 
 Minha idade é a raiz quadrada da raíz quadrada de 14 mais minha idade. 
 
 desenvolvendo achei: 
 
 x^4 - x - 14 = 0 
 
 encontrando as raízes por briot-rufini, achei 2. 
 
 Será que minha equaçao esta certa? 
 [...] 
 
Sim, mas o x não é o que você espera que ele seja -- o x é a raiz quarta 
da idade do indivíduo. Logo a idade é, na realidade, x^4 = 16. 
 
[]s, 
 
-- 
Fábio ctg \pi Dias Moreira 
 
= 
Instruções para entrar na lista, sair da lista e usar a lista em 
http://www.mat.puc-rio.br/~nicolau/olimp/obm-l.html 
= 
 
-- 

_
Voce quer um iGMail protegido contra vírus e spams? 
Clique aqui: http://www.igmailseguro.ig.com.br



Re: [obm-l] cococolegio navalvalval

2004-05-09 Por tôpico Fabio Henrique
Há um erro no final da solução ou estou equivocado? 
Se M^3=1000-3(10)-N^3, então M^3+N^3=970. 

Em  8 May 2004, [EMAIL PROTECTED] escreveu: 

Eu sei que estou sendo incoveniente mas essas 
quetões que eu venho mandando são duvidas de acumuladas 
de 2 anos,ficarei muito grato se voçês me ajudarem. 
 
 COLÉGIO NAVAL (1989) 
 
 Sendo M e N as raízes da equação X^2-10X+1=0 , o 
valor da expressão 1/M^3 + 1/N^2 é : 
 
 (A) 970 
 (B) 950 
 (C) 920 
 (D) 900 
 (E) 870 
=== 
 OBSERVAÇÃO: 
 
 Fazendo as raízes da equação ,encontraremos {5-2[6^ 
(1/2)]} e {5+2[6^(1/2)]} 
 
 fazendo de acordo com o enunciado teremos: 
 
 1 + 1 
  = 485+198(6^(1/2)) 
{5-2[6^(1/2)]}^3} {5+2[6^(1/2)]}^2} 
 
 Oque se aproxíma de 970,mas a resposta não é 
aproximada ,é exata e se voçê inverter os valores de M 
e N, o resultado será difernte. 
== 
 Fiz uma anologia más não ajudou muito 
 
 (M+N)^3=M^3+N^3+3(M^2)N+3(N^2)M 
 
 o que se conclui que : 
 
 M^3=(M+N)^3-3MN(M+N)-N^3 
 
 o que se conclui que: 
 
 M^3=1000-3(10)-N^3 
 M^3-n^3=970 
 O QUE SE TORNOU ESTRANHO É CONCLUIR QUE : 
 
 1/(m^3)+1/(n^2) = m^3-n^3 
=== 
 
__ 
Acabe com aquelas janelinhas que pulam na sua tela. 
AntiPop-up UOL - É grátis! 
http://antipopup.uol.com.br/ 
 
= 
Instruções para entrar na lista, sair da lista e usar a lista em 
http://www.mat.puc-rio.br/~nicolau/olimp/obm-l.html 
= 
 
-- 

_
Voce quer um iGMail protegido contra vírus e spams? 
Clique aqui: http://www.igmailseguro.ig.com.br



Re: [obm-l] COLEGIO NAVAL

2004-05-09 Por tôpico Fabio Henrique
Continuando... 

Há um problema que pede o lado em função do raio R e a resposta é 
L.sqrt(2)/2. 

Em 8 May 2004, [EMAIL PROTECTED] escreveu: 

leandro-epcar said: 
 COLÉGIO NAVAL (1987) 
 
 A equação do segundo grau X^2-2X+M=0,m0,tem raízes X' 
 e X ,se X'^(N-2)+X^(N-2)=A e X'^(N-1)+X^(N-1) 
 =B,então X'+X é: 
 (A) 2A+MB 
 (B) 2B-MA 
 (C) MA+2B 
 (D) MA-2B 
 (E) M(A-2B) 
 [...] 
 
Eu suponho que você quer X'^N + X^N? Caso contrário, é fácil ver que não 
há solução. Neste caso, se X e Y são as raízes (para simplificar a 
notação), temos que (x+y)(x^(n-1)+y^(n-1)) = x^n + y^n + 
xy(x^(n-2)+y^(n-2)), mas a gente sabe quanto valem x+y e xy. 
 
[]s, 
 
-- 
Fábio ctg \pi Dias Moreira 
 
= 
Instruções para entrar na lista, sair da lista e usar a lista em 
http://www.mat.puc-rio.br/~nicolau/olimp/obm-l.html 
= 
 
-- 

_
Voce quer um iGMail protegido contra vírus e spams? 
Clique aqui: http://www.igmailseguro.ig.com.br



[obm-l] Cadê?

2004-05-09 Por tôpico Fabio Henrique
Por onde andará Putinha da Silva? 

_
Voce quer um iGMail protegido contra vírus e spams? 
Clique aqui: http://www.igmailseguro.ig.com.br



Re: [obm-l] 8a.cone sul

2004-05-09 Por tôpico Fabio Henrique
Cara, não leva a mal. 
Você continua não colaborando com coisa alguma. Ou você faz observações que 
não acrescentam nada, ou faz este tipo de intervenção mágica. Isto é pouco 
instrutivo. O que são as equações de Pell? Por que você não explica para 
todos? Eu não sei o que são estas equações e resolvi. Acho que você é o 
maior enrolão. Pare de embromar e faça algo de construtivo como o Buffara, 
Steiner, Morgado, Nicolau, Santa Rita, Cyberhelp, etc. 
Tenho pena de quem vir a ser seu aluno. 
Passe bem. 



Em 8 May 2004, [EMAIL PROTECTED] escreveu: 

a=15*t^2-15t-995 
b=15*t^2-20t-992 
c=15*t^2-18t-993 
 
 --- Johann Peter Gustav Lejeune Dirichlet 
 escreveu:  E 
so usar umas equaçoes de Pell e o problema 
 sai. 
 A verdadeira treta ai e achar Todas e somente 
 TODAS as soluçoes. Encara essa! 
 
 --- [EMAIL PROTECTED] escreveu:  Essa é 
 boa.Demonstrar que existem infinitos 
  ternos (a,b,c),com a,b,c números 
  naturais,que satisfazem a 
  relação:2a^2+3b^2-5c^2=1997.Abraços vieira. 
  
  
 
_ 
  Voce quer um iGMail protegido contra vírus e 
  spams? 
  Clique aqui: 
 http://www.igmailseguro.ig.com.br 
  Ofertas imperdíveis! Link: 
  http://www.americanas.com.br/ig/ 
  
  
 
= 
  Instruções para entrar na lista, sair da 
 lista 
  e usar a lista em 
  
 
http://www.mat.puc-rio.br/~nicolau/olimp/obm-l.html 
  
 
= 
 
 
 = 
 TRANSIRE SVVM PECTVS MVNDOQVE POTIRI 
 
 CONGREGATI EX TOTO ORBE MATHEMATICI OB SCRIPTA 
 INSIGNIA TRIBVERE 
 
 Fields Medal(John Charles Fields) 
 
 N.F.C. (Ne Fronti Crede) 
 
 
 
 
__ 
 
 Yahoo! Messenger - Fale com seus amigos online. 
 Instale agora! 
 http://br.download.yahoo.com/messenger/ 
 
= 
 Instruções para entrar na lista, sair da lista 
 e usar a lista em 
 
http://www.mat.puc-rio.br/~nicolau/olimp/obm-l.html 
 
= 
 
= 
TRANSIRE SVVM PECTVS MVNDOQVE POTIRI 
 
CONGREGATI EX TOTO ORBE MATHEMATICI OB SCRIPTA INSIGNIA TRIBVERE 
 
Fields Medal(John Charles Fields) 
 
N.F.C. (Ne Fronti Crede) 
 
__ 
 
Yahoo! Messenger - Fale com seus amigos online. Instale agora! 
http://br.download.yahoo.com/messenger/ 
= 
Instruções para entrar na lista, sair da lista e usar a lista em 
http://www.mat.puc-rio.br/~nicolau/olimp/obm-l.html 
= 
 
-- 

_
Voce quer um iGMail protegido contra vírus e spams? 
Clique aqui: http://www.igmailseguro.ig.com.br



Re: [obm-l] 8a.cone sul

2004-05-09 Por tôpico Fabio Henrique
O recado foi para o Dirichlet. 


Em 9 May 2004, [EMAIL PROTECTED] escreveu: 

Em 9 May 2004, [EMAIL PROTECTED] escreveu: 
 
Cara, não leva a mal. 
Você continua não colaborando com coisa alguma. Ou você faz observações 
que 
não acrescentam nada, ou faz este tipo de intervenção mágica. Isto é pouco 
instrutivo. O que são as equações de Pell? Por que você não explica para 
todos? Eu não sei o que são estas equações e resolvi. Acho que você é o 
maior enrolão. Pare de embromar e faça algo de construtivo como o Buffara, 
Steiner, Morgado, Nicolau, Santa Rita, Cyberhelp, etc. 
Tenho pena de quem vir a ser seu aluno. 
Passe bem. 
 
Não é nada disso, sou muito esforçado e estou tentando aprender, se voçê 
não pode me ajudar não me desanime.Pena eu tenho de voçê por não me 
conhecer 
e já estabelecer um pré-conceito desses. 
 
Em 8 May 2004, [EMAIL PROTECTED] escreveu: 
 
a=15*t^2-15t-995 
b=15*t^2-20t-992 
c=15*t^2-18t-993 
 
 --- Johann Peter Gustav Lejeune Dirichlet 
 escreveu:  E 
so usar umas equaçoes de Pell e o problema 
 sai. 
 A verdadeira treta ai e achar Todas e somente 
 TODAS as soluçoes. Encara essa! 
 
 --- [EMAIL PROTECTED] escreveu:  Essa é 
 boa.Demonstrar que existem infinitos 
  ternos (a,b,c),com a,b,c números 
  naturais,que satisfazem a 
  relação:2a^2+3b^2-5c^2=1997.Abraços vieira. 
  
  
 
_ 
  Voce quer um iGMail protegido contra vírus e 
  spams? 
  Clique aqui: 
 http://www.igmailseguro.ig.com.br 
  Ofertas imperdíveis! Link: 
  http://www.americanas.com.br/ig/ 
  
  
 
= 
  Instruções para entrar na lista, sair da 
 lista 
  e usar a lista em 
  
 
http://www.mat.puc-rio.br/~nicolau/olimp/obm-l.html 
  
 
= 
 
 
 = 
 TRANSIRE SVVM PECTVS MVNDOQVE POTIRI 
 
 CONGREGATI EX TOTO ORBE MATHEMATICI OB SCRIPTA 
 INSIGNIA TRIBVERE 
 
 Fields Medal(John Charles Fields) 
 
 N.F.C. (Ne Fronti Crede) 
 
 
 
 
__ 
 
 Yahoo! Messenger - Fale com seus amigos online. 
 Instale agora! 
 http://br.download.yahoo.com/messenger/ 
 
= 
 Instruções para entrar na lista, sair da lista 
 e usar a lista em 
 
http://www.mat.puc-rio.br/~nicolau/olimp/obm-l.html 
 
= 
 
= 
TRANSIRE SVVM PECTVS MVNDOQVE POTIRI 
 
CONGREGATI EX TOTO ORBE MATHEMATICI OB SCRIPTA INSIGNIA TRIBVERE 
 
Fields Medal(John Charles Fields) 
 
N.F.C. (Ne Fronti Crede) 
 
__ 
 
Yahoo! Messenger - Fale com seus amigos online. Instale agora! 
http://br.download.yahoo.com/messenger/ 
= 
Instruções para entrar na lista, sair da lista e usar a lista em 
http://www.mat.puc-rio.br/~nicolau/olimp/obm-l.html 
= 
 
-- 
 
_ 
Voce quer um iGMail protegido contra vírus e spams? 
Clique aqui: http://www.igmailseguro.ig.com.br 
 
-- 
 
_ 
Voce quer um iGMail protegido contra vírus e spams? 
Clique aqui: http://www.igmailseguro.ig.com.br 
Ofertas imperdíveis! Link: http://www.americanas.com.br/ig/ 
 
= 
Instruções para entrar na lista, sair da lista e usar a lista em 
http://www.mat.puc-rio.br/~nicolau/olimp/obm-l.html 
= 
 
-- 

_
Voce quer um iGMail protegido contra vírus e spams? 
Clique aqui: http://www.igmailseguro.ig.com.br



Re: [obm-l] OT: antispam do UOL

2004-05-03 Por tôpico Fabio Henrique
Já que você tocou no assunto, não faz mesmo sentido entrar em uma lista de 
discussões com anti-spam ativado. 



Em 3 May 2004, [EMAIL PROTECTED] escreveu: 

favor desligarem seus antispams do UOL pois a cada mensagem que eu mando 
volta alguma coisa por causa dessa porcaria de antispam nada inteligente do 
UOL. 
 
[ ]'s 
 
= 
Instruções para entrar na lista, sair da lista e usar a lista em 
http://www.mat.puc-rio.br/~nicolau/olimp/obm-l.html 
= 
 
-- 

_
Voce quer um iGMail protegido contra vírus e spams? 
Clique aqui: http://www.igmailseguro.ig.com.br



Re: [obm-l] Conjectura de Goldbach

2004-04-29 Por tôpico Fabio Henrique
Apoiado! 



Em 29 Apr 2004, [EMAIL PROTECTED] escreveu: 

Olá, amigos. 
 
Entrei nessa lista há poucos dias, atraído pela possibilidade 
de ver boa matemática em ação, e eventualmente esclarecer 
dúvidas em análise funcional. 
 
Porém, depois de mensagens como a que o amigo 234 
respondeu, fico em dúvida quanto à credibilidade dessa lista. 
 
Temos um cara (o que usa MUITO injustamente a alcunha de 
Dirichlet) que só responde abobrinha e nunca resolve um 
exercício, um outro cara que acha que consegue conversar 
a respeito de relatividade como se estivesse num boteco com 
amigos, outro que manda oitenta mensagens por dia a respeito 
de integrais, e figuras menos marcantes, mas ainda assim risíveis. 
 
Em poucos dias por aqui, vi alguém enunciando a conjectura de 
Goldbach como se este fosse oriundo de um sanatório, um 
indivíduo querendo ficar craque em integrais mas não conseguindo 
resolver o problema da melancia, e por aí vai. 
 
Sinceramente, acho que vocês podem fazer melhor que isso se 
estudarem bastante e se dignarem a manter a boca fechada. 
Um abraço cordial a todos. 
 
Até a próxima, 
 
-- Gabriel 
 
- Original Message - 
From: [EMAIL PROTECTED] 
To: 
Sent: Wednesday, April 28, 2004 11:41 PM 
Subject: Re: [obm-l] Conjectura de Goldbach 
 
 Nossa, usou éter??? 
 
 Um número ímpar: 2k + 1 
 Outro número ímpar: 2n + 1 
 
 (2k + 1) + (2n + 1) = 2k + 2n + 2 
 
 = 2.(k + n + 1) = múltiplo de 2 = PAR 
 
 234 
 
 - Original Message - 
 From: Everton A. Ramos (www.bs2.com.br) 
 To: 
 Sent: Wednesday, April 28, 2004 10:57 PM 
 Subject: [obm-l] Conjectura de Goldbach 
 
 
  Boa noite... 
  
  Todo número par é a soma de dois números ímpares 
  
  ??? 
  
  Sedo X um número par... (X - 1) será ímpar... então (X - 1) + 1 = X 
  
  1 é ímpar, então... ??? 
  
  Porque isso é tão desafiante? 
 
= 
Instruções para entrar na lista, sair da lista e usar a lista em 
http://www.mat.puc-rio.br/~nicolau/olimp/obm-l.html 
= 
 
-- 

_
Voce quer um iGMail protegido contra vírus e spams? 
Clique aqui: http://www.igmailseguro.ig.com.br



Re: [obm-l] + 1 da sexta série...

2004-04-28 Por tôpico Fabio Henrique
Seja N o número de irmãos e M o número de irmãs. 
Cada filho tem N-1 irmãos e M irmãs == N-1 = M 
Cada filha tem N irmãos e M-1 irmãs == N = 2(M-1) 
Continue... 


Em 28 Apr 2004, [EMAIL PROTECTED] escreveu: 

Eu resolvi por um método meio louco, gostaria de saber se há resposta 
algébrica para essa coisa aqui: 
 
Um casal tem filhos e filhas. Cada filho tem o número de irmãos igual ao 
número de irmãs. Cada filha tem o número de irmãos igual ao dobro do número 
de irmãs. Qual o total de filhos e filhas do casal? 
 
Obs: Se alguém me ajudar a resolver esse aqui e o da melancia, dai vou 
poder entrar logo na geometria hiperbólica! 
 
Muito obrigado 
 
-- 

_
Voce quer um iGMail protegido contra vírus e spams? 
Clique aqui: http://www.igmailseguro.ig.com.br



Re: [obm-l] dúvidazinha!!!!!

2004-04-22 Por tôpico Fabio Henrique
Não deveria me meter nesta pendenga mas já estou a fazê-lo. 
Acho que a maioria das pessoas que andam participando da lista estão se 
acostumando a simplesmente perguntar antes de esgotar as suas próprias 
possibilidades. Este processo me parece totalmente estéril. 



Em 21 Apr 2004, [EMAIL PROTECTED] escreveu: 

quantos multiplos de 10, 5, 7 existem entre 1 e 
1000 ? 
 
-- 

_
Voce quer um iGMail protegido contra vírus e spams? 
Clique aqui: http://www.igmailseguro.ig.com.br



Re: [obm-l] dúvida

2004-04-03 Por tôpico Fabio Henrique
Observe que M^2 vale, no mínimo, zero. 
E M^2 + N vale, no mínimo, N (quando M=0) 
Pensemos então em f(x)=M^2+N. 
O menor valor que esta parábola assume é 3. Logo, N=3 (e isto acontece 
quando M=0. 
Uma função do segundo grau pode ser pensada sempre como 
f(x)= A.(x-p)^2 + q, onde q é o valor mínimo da função e p, o valor de x que 
proporciona este mínimo. A é aquele mesmo da forma A^2+Bx+C. 

Assim, o seu problema fica f(x)=A.(x-0)^2+3=A.x^2+3 
Observe que y=13 quando x=10. Substituíndo, encontrará A=1/10. 
Esta forma de escrever a função do 2º grau é chamada forma canônica. 
Abraços. 
Fabio Henrique 









um determinado fio é constituído de um material que, quando preso a dois 
 pontos distantes um do outro de 20m e ambos a 13m do solo, toma a forma 
de 
 uma parábola, estando o ponto mais baixo do fio a 3m do solo. Assinale a 
 alternativa que corresponde à parábola no sistema de coordenadas 
 cartesianas XOY, onde o eixo OY contém o ponto mais baixo do fio e o eixo 
 OX está sobre o solo. 
 [...] 
 
Inicialmente, note que fios suspensos *não* formam parábolas, mas sim 
catenárias, que são coisas parecidas com o gráfico de (e^x + e^-x)/2. 
 
Pedantismos físicos à parte, note que se os pontos de apoio são os pontos 
(10, 
13) e (-10, 13), então o ponto mais baixo é o ponto (0, 3). Associe à 
parábola uma função f(x) tal que todos os pontos da parábola são da forma 
(x, 
f(x)). Portanto, f(10) = f(-10) = 13 e f(0) = 3. 
 
Obviamente, f é quadrática. Considere g(x) = f(x) - 13. Então 10 e -10 são 
dois zeros de g, logo g(x) = a*(x-10)*(x+10), onde a é um real. Como g(0) = 
- -10, a*(-10)*10 = -10 = a = 1/10. Logo g(x) = (x-10)*(x+10)/10, logo 
f(x) = 
(x-10)*(x+10)/10 + 13. Como as alternativas da questão não chegaram aqui, 
essa é a melhor resposta que eu posso dar. 
 
[]s, 
 
- -- 
Fábio Dias Moreira 
http://dias.moreira.nom.br/ 
-BEGIN PGP SIGNATURE- 
Version: GnuPG v1.2.3 (GNU/Linux) 
 
iD8DBQFAb1YbalOQFrvzGQoRAmytAKDAnMNnuzn97j+I85/F8k+fOiM2xACfbeon 
dRZpXPhtXJ8ltTc/tINuXZQ= 
=bpms 
-END PGP SIGNATURE- 
 
= 
Instruções para entrar na lista, sair da lista e usar a lista em 
http://www.mat.puc-rio.br/~nicolau/olimp/obm-l.html 
= 
 
-- 

_
Voce quer um iGMail protegido contra vírus e spams? 
Clique aqui: http://www.igmailseguro.ig.com.br



[obm-l] Re: Equaçoes irracionaiis

2004-03-27 Por tôpico Fabio Henrique
Faltou dizer porque: 

sqrt vem de square root. 



Em 27 Mar 2004, [EMAIL PROTECTED] escreveu: 

Please, qual nomenclatura eu devo utilizar para raíz quadrada e divisão? 
Em quanto nao sei, vou apenas escreve-la. 
Me ajudem a resolver esta questao: 
(Raíz Q x^2+9) - {15 / (Raíz Q x^2+9 ) }= 2 
 
_ 
MSN Messenger: converse com os seus amigos online. 
http://messenger.msn.com.br 
 
Instruções 
para entrar na lista, sair da lista e usar a lista em 
http://www.mat.puc-rio.br/~nicolau/olimp/obm-l.html 
 
 
-- 

_
Voce quer um iGMail protegido contra vírus e spams? 
Clique aqui: http://www.igmailseguro.ig.com.br



Re: [obm-l] livros

2004-03-20 Por tôpico Fabio Henrique
Não é verdade. Eu VI E FOLHEEI um esta semana na Galileu do L. Machado. 


Em 18 Mar 2004, [EMAIL PROTECTED] escreveu: 

Olá Antônio, 
 
 Por acaso a livraria é: 
 Livraria Academia do Saber 
 Av. Passos, 25 
Centro 
Tel: (21) 2242-4826 
Livros novos e usados 
 
 http://www.aosaber.com.br/ 
 
 Infelizmente, esse site está desativado. 
 Espero estar errado... se você tiver o endereço eletrônico, eu agradeço. 
 []´s Nelson 
 
 p.s.: Realmente, no site da livraria galileu consta os livros, mas eles 
acabaram de me avisar que eles estão na verdade esgotados. 
 
 Antonio Neto wrote: 
 
 Por acaso, soube no sábado passado. Se voce estiver no Rio de Janeiro, 
encontrarah uma livraria na Av. Passos, 23 ou 25, mas estah de frente para a 
rua. Encontrei 5 ou 6 exemplares de cada, em bom estado. Sem querer imitar 
nenhum participante da lista, depois eu mando o endereco eletronico, eh que 
estou no trabalho. Abracos, olavo. 
 
 From: Nelson 
 Reply-To: [EMAIL PROTECTED] 
 To: [EMAIL PROTECTED] 
 Subject: [obm-l] livros 
 Date: Wed, 17 Mar 2004 11:36:14 -0300 (ART) 
  
 Olá a todos, 
  
 Alguém poderia me dizer como encontro os livros: 
 GEOMETRIA I e II, e ALGEBRA I (morgado, a.c., et alii) 
  
 E alguém sabe como entrar em contato com a editora Francisco Alves? 
  
 Desde já, agradeço. 
 Nelson 
 
-- 

_
Voce quer um iGMail protegido contra vírus e spams? 
Clique aqui: http://www.igmailseguro.ig.com.br



Re: [obm-l] Pequeno erro (um desafio em calculadoras)

2004-03-20 Por tôpico Fabio Henrique
Rafael. 
Em primeiro lugar, não concordo que o Niski tenha errado o problema por 
causa da aproximação. Ele sabia que 
i = 4log[2](10) e 
12  4log[2](10) 16 
Isto NÃO QUER DIZER que i = 16. O i pode estar entre 4log[2](10)e 16. 
Em segundo lugar, pode ser enfadonho calcular a raiz cúbica de um número em 
uma calculadora com 4 operações e raiz quadrada, se tivermos que recorrer a 
isso todas as vezes que este cálculo aparecer. No entanto, acho um desafio 
interessantíssimo e muito instrutivo no que diz respeito aos métodos 
iterativos de resoluções de equações. Pode ser um exemplo interessante para 
se começar uma aula sobre Método de Newton. 
Saudações matemáticas. 
Fabio Henrique 

Em 19 Mar 2004, [EMAIL PROTECTED] escreveu: 

Bem, o fato é que eu não sugeri em *nenhum* momento que se usasse uma 
calculadora. A minha idéia era simplesmente mostrar que a aproximação feita 
do log(10,2) estava gerando o erro. Há pouco li uma mensagem do Ricardo em 
que ele expôs habilmente como a aproximação estava errada, algo que eu não 
havia feito. 
 
Sobre o problema que você propõe, bem, ele é muito enfadonho. Antes das 
calculadoras, foram criados algoritmos para a extração de raízes quadradas 
e 
cúbicas. Eu cheguei a aprender ambos, embora isso só fizesse sentido se 
ensinado há algumas décadas, e não sou tão jurássico assim. O algoritmo 
para 
a extração de raízes quadradas é até útil, se você quiser uma aproximação 
de 
até 2 casas decimais. Para mais do que isso, você já precisará de uma 
calculadora para realizar as multiplicações gigantescas, subtrações 
intermediárias etc. Suponho que você tenha se referido ao das raízes 
cúbicas, mas este é ainda pior nas operações, mesmo que, obviamente, deva 
ter sido válido mito tempo atrás, embora eu não ache propriamente algo 
prático ou divertido... 
 
- Original Message - 
From: Johann Peter Gustav Lejeune Dirichlet 
 
To: 
Sent: Friday, March 19, 2004 6:43 PM 
Subject: Re: [obm-l] Pequeno erro (um desafio em calculadoras) 
 
A ideia, Rafael, e que nao e muito logico (nao a 
mentes humanas...) usar a calculadora em apenas 
uma conta do problema, quando o problema inteiro 
pode ser feito com uma boa calculadora. 
Para rechear a mensagem proponho o problema: 
 
com uma calculadora que tenha as operaçoes 
basicas determine a raiz cubica de um numero dado 
(e claro que nao exijo exatidao de 100 por cento 
mas de pelo menos todas as cifras da calculadora, 
o que da 8 ou 10 digitos).Pode ser um metodo 
iterativo (ter uma aproximaçao e melhora-la, 
usando a mesma receita). 
Acho isso facil mas e so para diversao mesmo.. 
 
= 
Instruções para entrar na lista, sair da lista e usar a lista em 
http://www.mat.puc-rio.br/~nicolau/olimp/obm-l.html 
= 
 
-- 

_
Voce quer um iGMail protegido contra vírus e spams? 
Clique aqui: http://www.igmailseguro.ig.com.br



[obm-l] Olimpíada brasileira

2004-03-17 Por tôpico Fabio Henrique
A Olimpíada Brasileira de Matemática para ensino médio e fundamental já tem 
data? Como posso fazer para inscrever os alunos do colégio onde trabalho? 
Obrigado. 
Fabio Henrique 

_
Voce quer um iGMail protegido contra vírus e spams? 
Clique aqui: http://www.igmailseguro.ig.com.br



[obm-l] Re: dúvida

2004-03-17 Por tôpico Fabio Henrique
L = V-C = 20%deV = V/5 
Assim, C = 4V/5 

L/C = (V/5)/(4V/5) = 1/4 

So que estes 20% sobre o preco de venda chamam-se MARGEM DE LUCRO. O que 
chamamos LUCRO eh o percentual sobre o preco de custo. 



Em 17 Mar 2004, [EMAIL PROTECTED] escreveu: 

olá amigos estou com uma dúvida; 
 
 O lucro obtido na venda de certo artigo corresponde 
a 20% de seu preço de venda. A razão entre os valores que correspondem ao 
lucro e ao preço de custo desse artigo pode ser expressa pela 
fração: 
 
 a)1/4 b)1/5 c)4/5 d)5/6 e)6/5 
 
-- 

_
Voce quer um iGMail protegido contra vírus e spams? 
Clique aqui: http://www.igmailseguro.ig.com.br



Re: [obm-l] En: Putnam Question

2004-03-14 Por tôpico Fabio Henrique
Encontrei (EC.TS.TR)/(BC.AE.TC). 
Confere? 


Em 12 Mar 2004, [EMAIL PROTECTED] escreveu: 

Sauda,c~oes, 
 
Um de geometria de uma outra lista. 
 
[]'s 
Luis 
 
-Mensagem Original- 
De: Ben @hotmail.com 
Para: 
Enviada em: segunda-feira, 9 de fevereiro de 2004 17:31 
Assunto: Putnam Question 
 
 This is a problem I found in the 2001 Putnam exam. Anyone want to take 
 a stab at it? 
 
 Triangle ABC has an area 1. Points E, F, and G,lie respectively on 
 sides BC , CA, and AB, such that AE bisects BF at point R, BF bisects 
 CG at point S, and CG bisects AE at point T, Find the area of triangle 
 RST. 
 
 And go 
 
 
= 
Instruções para entrar na lista, sair da lista e usar a lista em 
http://www.mat.puc-rio.br/~nicolau/olimp/obm-l.html 
= 
 
-- 

_
Voce quer um iGMail protegido contra vírus e spams? 
Clique aqui: http://www.igmailseguro.ig.com.br



Re: [obm-l] Livros de quimica e fisica para provas do ITA/IME

2004-03-14 Por tôpico Fabio Henrique
Eu sugeriria os do Feltre com o SETSUO YOSHINAGA. 

Você só vai encontrar em lojas de livros usados. 



Em 13 Mar 2004, [EMAIL PROTECTED] escreveu: 

Apesar dessa não ser a lista ideal para fazer estas perguntas foi a que 
mais se aproximou, então la vai: 
Quais são os livros de quimica e fisica mais indicado para quem vai 
prestar ITA e IME? 
Atenciosamente, 
Allan 
 
= 
Instruções para entrar na lista, sair da lista e usar a lista em 
http://www.mat.puc-rio.br/~nicolau/olimp/obm-l.html 
= 
 
-- 

_
Voce quer um iGMail protegido contra vírus e spams? 
Clique aqui: http://www.igmailseguro.ig.com.br



[obm-l] Re: Res: [off-topic] Pentagono regular

2004-03-14 Por tôpico Fabio Henrique
Parece-me que a indignação do Rafael dá-se contra os pareceres pouco 
esclarecedores do Dirichlet. 

Em 14 Mar 2004, [EMAIL PROTECTED] escreveu: 

On Sun, Mar 14, 2004 at 05:35:46PM -0300, Rafael wrote: 
 Concordo em absoluto com tudo o que você expôs, Artur. Discordo 
 veementemente de outra coisa: apelar para a Trigonometria quando não se 
 enxerga algo mais simples em Geometria. Geralmente, as soluções 
 trigonométricas nada têm de elegantes e, para muitos casos, embora 
resolvam 
 o problema, dão um trabalho muito maior do que se utilizássemos conceitos 
de 
 Geometria. A verdade é que a Geometria exige uma percepção de construções 
 que a Trigonometria, de certa forma, dispensa. 
 
Peço a todos um pouco mais de calma ao discutir algo que é puro gosto: 
o Dirichlet gosta de soluções trigonométricas e o Rafael não. 
Tudo bem que cada um queira dar a sua opinião neste debate milenar, 
mas sem esquentar demais. 
 
A minha impressão como professor é que existem claras diferenças entre 
indivíduos: uns tem mais facilidade para um tipo de solução, outros 
tem mais facilidade com outro tipo. O bom estudante de olimpíada conhece 
os seus próprios pontos fortes e fracos e sabe qual a melhor estratégia 
para ele. Ele também sabe que não deve desprezar os métodos que não 
são especialmente do gosto dele. 
 
[]s, N. 
= 
Instruções para entrar na lista, sair da lista e usar a lista em 
http://www.mat.puc-rio.br/~nicolau/olimp/obm-l.html 
= 
 
-- 

_
Voce quer um iGMail protegido contra vírus e spams? 
Clique aqui: http://www.igmailseguro.ig.com.br



[obm-l] Re: Seqüência

2004-03-08 Por tôpico Fabio Henrique
312211 
e 
13112221 



Em 8 Mar 2004, [EMAIL PROTECTED] escreveu: 

Eis um problema muito interessante: 
 
Seja a seqüência numérica (1, 11, 21, 1211, 111221, ...), quais são o sexto 
e sétimo termos? 
 
Abraços, 
 
Rafael de A. Sampaio 
 
= 
Instruções para entrar na lista, sair da lista e usar a lista em 
http://www.mat.puc-rio.br/~nicolau/olimp/obm-l.html 
= 
 
-- 

_
Voce quer um iGMail protegido contra vírus e spams? 
Clique aqui: http://www.igmailseguro.ig.com.br



[obm-l] Re: ú]¾'Z

2004-03-07 Por tôpico Fabio Henrique
Já que você perguntou, se não estou enganado, alguém falou que a bandeira do 
Brasil tem DOIS retângulos, um losango e um círculo. 
??? 


 Em 07 Mar 2004, [EMAIL PROTECTED] escreveu: 

Re: [obm-l] dúvida 
 
on 07.03.04 16:35, TSD at [EMAIL PROTECTED] wrote: 
 
 OLÁ AMIGOS ESTOU COM UMA PEQUENA DÚVIDA. DE QUANTAS MANEIRAS EU POSSO 
COLORIR A BANDEIRA DO BRASIL DE MODO QUE AS REGIÕES COMUNS(ÁREA DELIMITIDA 
PELAS FIGURAS) SEJAM PINTADAS COM CORES DIFERENTE. 
 
AGUARDO RESPOSTA 
 
Esse problema tah mal definido. E as estrelas? Tem que ser pintadas tambem? 
 
E a inscricao ORDEM E PROGRESSO? Pode ser cada letra duma cor? 
 
-- 

_
Voce quer um iGMail protegido contra vírus e spams? 
Clique aqui: http://www.igmailseguro.ig.com.br



[obm-l] Tri isósceles

2004-02-29 Por tôpico Fabio Henrique
Problema do triângulo isósceles 

Vamos imaginar que A=20º, AB=AC=1. Considere E sobre AC e F sobre AB. 
CBE=60º e BCF=50º 
O ângulo procurado é BEF 
De saída, sabemos que: 
EBF=20º 
FCE=30º 
BFC=50º 
BEC=40º 
Desconfiando que os triângulos AFC e BFE fossem semelhantes, resolvi tentar 
provar o fato. 

Aplicando Lei dos Senos no triângulo ABC temos BC=sen20/sen80 
Aplicando Lei dos Senos no triângulo ABE temos BE=sen20/sen40 
Aplicando Lei dos Senos no triângulo ABC temos AF=sen30/sen50 
Além disso, BF=1-(sen30/sen50) 

AF/AC=sen30/sen50 

BF/BE=[1-(sen30/sen50)]/(sen20/sen40) 
BF/BE=[(sen50-sen30)/sen50)]/(sen20/sen40) 

Neste ponto, precisamos observar que sen50=sen40.cos10+sen10.cos40 
   e sen10=sen40.cos10-sen10.cos40 
Logo, sen50-sen10=2.sen10.cos40 
Então BF/BE=[2.sen10.cos40/sen50]/[sen20/sen40] 
BF/BE=2.sen10.cos40.sen40/sen50.sen20 
BF/BE=sen10.sen80/sen50.sen20 
BF/BE=sen10.cos10/sen50.sen20 
BF/BE=sen20/2.sen50.sen20 
BF/BE=0,5/sen50 
BF/BE=sen30/sen50=AF/AC 

Isto garante que os triângulos AFC e BFE são semelhantes. 
Assim med(ACF)=med(BEF)=30º 

SENHORES, DÊEM UMA OLHADA E CONFIRAM SE ME EQUIVOQUEI EM ALGUMA PASSAGEM. 

[]'s 

Fabio Henrique. 

_
Voce quer um iGMail protegido contra vírus e spams? 
Clique aqui: http://www.igmailseguro.ig.com.br



Re: [obm-l] Problema de quadrado perfeito

2004-02-28 Por tôpico Fabio Henrique
Seja x = k^2 e x+99 = p^2 
Desta forma, k^2 +99 = p^2 
 p^2 - k^2 = 9 x 11 
 (p-k)(p+k)= 9 x 11 
Assim, p=10 e k=1 ou p=-10 e k=-1 
Logo, x=1. 

Em 28 Feb 2004, [EMAIL PROTECTED] escreveu: 

1 ) Quantos inteiros positivos x são 
tais que tanto x quanto x+ 99 são quadrados perfeitos? 
 
 Eu nao entendi bem o enunciado.. quem puder da uma 
explicada ae eu agradeço! 
 
 Um abraço!!! 
 
-- 

_
Voce quer um iGMail protegido contra vírus e spams? 
Clique aqui: http://www.igmailseguro.ig.com.br



Re: [obm-l] Diedros e Triedros

2004-02-28 Por tôpico Fabio Henrique
Em todo triedro, qualquer face é menor que a soma das outras duas. 
|B - C|  A  B + C 

Seja um triedro v(a,b,c). Vamos supor que ac é a maior face(ou ângulo de 
face) de V. 

Construa em ac um ângulo b'c congruente a bc. 
Tome um ponto B'em b' de forma que o segmento VB seja congruente ao segmento 
VB'. 
Considere uma seção ABC (A em VA). 
Os triângulos BVB e B'VC são congruentes. Conclui-se que BC congr. a B'C. 
No triângulo ABC, temos ACAB+BC = AB'+B'CAB+BC = AB'AB. 
Considere os triângulos B'VA e BVA. Então, ab' ab. 

ab'  ab 
b'c = bc 

então ab' + b'c  ab + bc. 

Grande abraço. Fabio Henrique. 



Em 27 Feb 2004, [EMAIL PROTECTED] escreveu: 

Boa noite aos colegas da lista. 
 
Há pouco tempo, estava eu estudando diedros, triedros, poliedros e ângulos 
poliédricos. O livro que estava lendo afirmava verdadeiras algumas 
desigualdades, mas não as demonstrava. Assim, fiquei sem saber se 
realmente elas são intuitivamente verificadas somente ou se existe uma 
demonstração formal, que ainda não encontrei. 
 
Sejam A, B e C faces e d_1, d_2 e d_3 diedros (ângulos entre faces), as 
afirmações são: 
 
A soma das medidas (em graus) das faces de um triedro qualquer é menor que 
360º. 
 
0º  A + B + C  360º 
 
Em todo triedro, qualquer face é menor que a soma das outras duas. 
 
|B - C|  A  B + C 
 
A soma dos diedros de um triedro está compreendida entre 2 retos e 6 
retos. 
 
180º  d_1 + d_2 + d_3  540º 
 
Em qualquer triedro, a medida (em graus) de um diedro aumentada de 180º 
supera a soma das medidas dos outros dois. 
 
d_1 + 180º  d_2 + d_3 
 
Fica o meu agradecimento desde já a quem puder demonstrar, comentar ou 
quaisquer referências que possam ser consultadas sobre o assunto. 
 
Abraços, 
 
Rafael de A. Sampaio 
 
= 
Instruções para entrar na lista, sair da lista e usar a lista em 
http://www.mat.puc-rio.br/~nicolau/olimp/obm-l.html 
= 
 
-- 

_
Voce quer um iGMail protegido contra vírus e spams? 
Clique aqui: http://www.igmailseguro.ig.com.br



Re: [obm-l] Diedros e Triedros

2004-02-28 Por tôpico Fabio Henrique
A soma das medidas (em graus) das faces de um triedro qualquer é menor que 
360º. 

0º  A + B + C  360º 

Considere a semi-reta Va’ oposta a Va. No triedro V(a’,b,c) temos bc  ba’+ 
ca’. 
Observe que ab + ba’ = 180 e ac + ca’ = 180 
Então, ab + ba’ +  ac + ca’ =  360 
Como bc  ba’+ ca’, ta provado. 

Outro abraço. 
Fabio Henrique. 



Em 27 Feb 2004, [EMAIL PROTECTED] escreveu: 

Boa noite aos colegas da lista. 
 
Há pouco tempo, estava eu estudando diedros, triedros, poliedros e ângulos 
poliédricos. O livro que estava lendo afirmava verdadeiras algumas 
desigualdades, mas não as demonstrava. Assim, fiquei sem saber se 
realmente elas são intuitivamente verificadas somente ou se existe uma 
demonstração formal, que ainda não encontrei. 
 
Sejam A, B e C faces e d_1, d_2 e d_3 diedros (ângulos entre faces), as 
afirmações são: 
 
A soma das medidas (em graus) das faces de um triedro qualquer é menor que 
360º. 
 
0º  A + B + C  360º 
 
Em todo triedro, qualquer face é menor que a soma das outras duas. 
 
|B - C|  A  B + C 
 
A soma dos diedros de um triedro está compreendida entre 2 retos e 6 
retos. 
 
180º  d_1 + d_2 + d_3  540º 
 
Em qualquer triedro, a medida (em graus) de um diedro aumentada de 180º 
supera a soma das medidas dos outros dois. 
 
d_1 + 180º  d_2 + d_3 
 
Fica o meu agradecimento desde já a quem puder demonstrar, comentar ou 
quaisquer referências que possam ser consultadas sobre o assunto. 
 
Abraços, 
 
Rafael de A. Sampaio 
 
= 
Instruções para entrar na lista, sair da lista e usar a lista em 
http://www.mat.puc-rio.br/~nicolau/olimp/obm-l.html 
= 
 
-- 

_
Voce quer um iGMail protegido contra vírus e spams? 
Clique aqui: http://www.igmailseguro.ig.com.br



Re: [obm-l] Ops! Problema de quadrado perfeito

2004-02-28 Por tôpico Fabio Henrique
Claro! Eu dei uma mongolizada incrível! 

Veja só. Quando quebrei o 99, o fiz de uma só maneira: 9 x 11. 
No entanto, isto pode ser feito de TRÊS modos: 9x11, 3x33 e o trivial 1x99. 
A primeira nós já fizemos. Vamos aos outros: 
2º) p+k=33 e p-k=3 = p=18 e k=15 (O valor de x é 225) 
3º) p+k=99 e p-k=1 = p=50 e k=49 (O valor de x é 2401). 

Desculpe-me o equívoco. 




Em 28 Feb 2004, [EMAIL PROTECTED] escreveu: 

Opa... tipo, entendi mais ou menos seu raciocinio.. 
 
mas o gabarito é 3. eu tbm tinha achado 1... mas errei. 
 
hmm ... 
- Original Message - 
From: Fabio Henrique 
To: 
Sent: Saturday, February 28, 2004 11:00 AM 
Subject: Re: [obm-l] Problema de quadrado perfeito 
 
 Seja x = k^2 e x+99 = p^2 
 Desta forma, k^2 +99 = p^2 
 p^2 - k^2 = 9 x 11 
 (p-k)(p+k)= 9 x 11 
 Assim, p=10 e k=1 ou p=-10 e k=-1 
 Logo, x=1. 
 
 Em 28 Feb 2004, [EMAIL PROTECTED] escreveu: 
 
 1 ) Quantos inteiros positivos x são 
 tais que tanto x quanto x+ 99 são quadrados perfeitos? 
  
  Eu nao entendi bem o enunciado.. quem puder da uma 
 explicada ae eu agradeço! 
  
  Um abraço!!! 
  
 -- 
 
 _ 
 Voce quer um iGMail protegido contra vírus e spams? 
 Clique aqui: http://www.igmailseguro.ig.com.br 
 
 
 Esta mensagem foi verificada pelo E-mail Protegido Terra. 
 Scan engine: VirusScan / Atualizado em 27/02/2004 / Versão: 1.4.1 
 Proteja o seu e-mail Terra: http://www.emailprotegido.terra.com.br/ 
 
 E-mail classificado pelo Identificador de Spam Inteligente Terra. 
 Para alterar a categoria classificada, visite 
 
http://www.terra.com.br/centralunificada/emailprotegido/imail/imail.cgi?+_u=fabiocontreiras_l=1077977988.52812.13512.turvo.terra.com.br
 
 
 
= 
Instruções para entrar na lista, sair da lista e usar a lista em 
http://www.mat.puc-rio.br/~nicolau/olimp/obm-l.html 
= 
 
-- 

_
Voce quer um iGMail protegido contra vírus e spams? 
Clique aqui: http://www.igmailseguro.ig.com.br



Re: RE: [obm-l]

2004-02-25 Por tôpico Fabio Henrique
PRIMEIRA QUESTÃO: 

Vamos tentar arrumar a expressão x^2+3xy+y^2 dentro de alguma outra que 
tenha vindo diretamente de 2x+y. 

2x+y=1 
(2x+y)^2=1 
(2x+y)^2=4x^2+4xy+y^2=(x^2+3xy+y^2)+3x^2+xy=1 
(2x+y)^2=(x^2+3xy+y^2)+x.(3x+y)=1 
(2x+y)^2=(x^2+3xy+y^2)+x.(x+2x+y)=1 
(2x+y)^2=(x^2+3xy+y^2)+x.(x+1)=1 
(x^2+3xy+y^2)=1-x-x^2 
Calcule agora o maior valor de -x^2-x+1 (use -(delta)/4a) 


24 Feb 2004, [EMAIL PROTECTED] escreveu: 

Eu acho que consegui a terceira. 
 
 (ab)^2 = b^2 + 10ab + 10ab + 100a^2 (I) 
 (ba)^2 = a^2 + 10ab + 10ab + 100b^2 (II) 
 
 fazendo (I) - (II) - 99a^2 - 99b^2 
 
(cc)^2= c^2 + 20c^2 +100c^2 
 
99a^2 - 99b^2 = 121c^2 - 9a^2-9b^2 = 11c^2 
 
 9(a^2-b^2) = 11c^2 - como 11 eh primo, temos c^2 múltiplo de nove, 
ou c=0,3,6 ou 9 
 
 fazendo c=3, 9(a^2-b^2) = 11.9 - a^2-b^2 = 11 
 
 por tentativa e erro temos a=6 e b = 5 
 
 65^2 - 56^2 = 33^2 
 
OLÁ AMIGOS ESTOU COM MAIS ALGUMAS DÚVIDAS. PODERIAM AJUDAR POR FAVOR. 
 
1) Se 2x + y = 1, com x e y reais, então o maior valor da expressão x² 
+ 3xy + y² é igual a ; 
A)5/4 B)7/4 C)13/8 D)17/8 E)31/16 
 
2) Se x e y são números inteiros e positivos, representa-se o máximo 
divisor comum de x e y por mdc (x,y); assim, o número de pares ordenados 
(x,y) que são soluções do sistema : 
 x + y = 810 
mdc(x,y)E 
 
A)6 B)8 C)10 D)16 E)18 
 
3) Se a, b e c são algarismos distintos, no sistema de numeração 
decimal existe um único número de dois algarismos (ab) tal que (ab)² - 
(ba)² = (cc)². 
O valor de (a + b + c) é igual a: 
A) 11 B)12 C)13 D)14 E)15 
 
 
_ 
MSN Messenger: converse com os seus amigos online. 
http://messenger.msn.com.br 
 
Instruções 
para entrar na lista, sair da lista e usar a lista em 
http://www.mat.puc-rio.br/~nicolau/olimp/obm-l.html 
 
 
-- 

_
Voce quer um iGMail protegido contra vírus e spams? 
Clique aqui: http://www.igmailseguro.ig.com.br



Re: [obm-l] duvidazinha

2004-02-23 Por tôpico Fabio Henrique
D=d.q+r 
3x^2 + 4 = x.3x + x-1 
3x^2 + 4 = 3x^2 + x-1 
4 = x-1 
x = 5 



Em 22 Feb 2004, [EMAIL PROTECTED] escreveu: 

\OLÁ COLEGAS , PODERIAM AJUDAR NESTE PROBLEMA: 
 
 1) numa divisão, o dividendo é igual a 3x²+4, o divisor é 
igual a x, o quociente é o triplo do divisor e o resto é o maior possível. 
O 
número que corresponde `a soma do dividendo com o resto é igual a? 
 
-- 

_
Voce quer um iGMail protegido contra vírus e spams? 
Clique aqui: http://www.igmailseguro.ig.com.br



[no subject]

2004-02-23 Por tôpico Fabio Henrique
SENHORES, O QUE HÁ COM VOCÊS? 

A segunda progressão não é uma progressão geométrica. 

2 + 1 +1/2 +... = 3 ? 

Errar é normal mas vamos ter mais cuidado. 

[]'s 
Fabio Henrique. 

Junior, 

1 + 2/2 + 3/4 + 4/8 + 5/16 + ... = 1 + 1 + 3/4 + 1/2 + 5/16 + ... = 1 + 
(1 + 1/2 + ...) + (3/4 + 5/16 + ...) 

Calculando o limite da soma para a primeira progressão, sabendo-se que a 
razão é 1/2 e o primeiro termo é 1: 

1/(1-1/2) = 2 

Calculando o limite da soma para a segunda progressão, sabendo-se que a 
razão é 5/16 / 3/4 = 5/12 e o primeiro termo é 3/4: 

3/4/(1-5/12) = 9/7 

Somando-se: 1 + 2 + 9/7 = 30/7 é o limite da soma dos infinitos termos para 
a P.G. inicial. 



faz o seguinte: 

1 = 1 
2/2 = 1/2 +1/2 
3/4 = 1/4 + 1/4 +1/4 

e assim sucessivamente 

soma coluna por coluna, 
então vc vai ter q 

1+2/2+3/4+4/8+5/16... = (1+1/2+...)+ (1/2 +1/4+..) + 
(1/4+ 1/8+...) +... 

= 2 + 1 +1/2 +... = 3 

_
Voce quer um iGMail protegido contra vírus e spams? 
Clique aqui: http://www.igmailseguro.ig.com.br



Re: [obm-l] torres

2004-02-20 Por tôpico Fabio Henrique
Pode pensar com as rainhas... 


Em 20 Feb 2004, [EMAIL PROTECTED] escreveu: 

Ola, 
 
Tem certeza que digitou corretamente o enunciado ? 
 
Seria: 
 
De forma que 2 torres nao estejam na mesma linha 
 
OU 
 
De forma que as 8 torres nao se ataquem ? 
 
Ps: Eu ja vi um bem interessante: Coloque 8 rainhas em um tabuleiro sem que 
nenhuma ataque as outras  
 
Em uma mensagem de 19/2/2004 17:33:00 Hora padrão leste da Am. Sul, 
[EMAIL PROTECTED] escreveu: 
 
De quantas maneiras podemos arrumar 8 torres iguais em um tabuleiro de 
forma 
 
que duas torres não estejam na mesma linha, coluna ou diagonal? 
 
-- 

_
Voce quer um iGMail protegido contra vírus e spams? 
Clique aqui: http://www.igmailseguro.ig.com.br



Re: [obm-l] Orgia de livros

2004-02-19 Por tôpico Fabio Henrique
Só não acho interessante vincular à nossa lista algo ilegal. Façam por fora. 
Não nos envolvam. 


Em 19 Feb 2004, [EMAIL PROTECTED] escreveu: 

Gostei do link. Muito bom. 
 Se a pessoa está ou não fora da lei eu não 
sei, mas não me importo hahaha... 
 Mas o fato é que eu estou ávido de conhecimento como 
Tartaglia (que furtou livros para aprender álgebra). 
 e já baixei tudo pra uma pasta no 
meu computador (fiz isso antes da página sair do ar). 
 
 VALEU BROTHER 
 
[]s Ronaldo L. Alonso 
 
Parece que esta página é ilegal... 
 
Muitos livros colocados lá tem os seus direitos protegidos, como o 
Concrete 
Mathematics. Outros foram temporariamente disponibilizados na internet por 
seus autores, como os do Wilf. 
 
Não sei quem mantém a página, mas perante as leis de direitos autorais, 
esta 
pessoa está cometendo um crime. 
 
Abraços, Paulo 
 
 
_ 
Voce quer um iGMail protegido contra vírus e spams? 
Clique aqui: http://www.igmailseguro.ig.com.br 
Ofertas imperdíveis! Link: http://www.americanas.com.br/ig/ 
 
= 
Instruções para entrar na lista, sair da lista e usar a lista em 
http://www.mat.puc-rio.br/~nicolau/olimp/obm-l.html 
= 
 
-- 

_
Voce quer um iGMail protegido contra vírus e spams? 
Clique aqui: http://www.igmailseguro.ig.com.br



[obm-l] torres

2004-02-19 Por tôpico Fabio Henrique
De quantas maneiras podemos arrumar 8 torres iguais em um tabuleiro de forma 
que duas torres não estejam na mesma linha, coluna ou diagonal? 

_
Voce quer um iGMail protegido contra vírus e spams? 
Clique aqui: http://www.igmailseguro.ig.com.br



Re: [obm-l] Ajuda

2004-02-15 Por tôpico Fabio Henrique
Depende do teu propósito. 


Em 14 Feb 2004, [EMAIL PROTECTED] escreveu: 

Gostaria da ajuda dos amigos. 
 
 Qual destes dois livros de geometria plana é 
melhor? 
 Os livros de geometria do Morgado e do Eduardo 
Wagner(Geometria1 e 2) 
 
 Ou o Fundamentos da Matemática Volume 
9 
 
-- 

_
Voce quer um iGMail protegido contra vírus e spams? 
Clique aqui: http://www.igmailseguro.ig.com.br



Re: [obm-l] Compras

2004-02-15 Por tôpico Fabio Henrique
A moto custa, à vista, 8000.(1-x) e este é o valor de que dispõe Paulo. 
Deste valor, Paulo tira 4000 para pagar a 1ª. O restante [8000.(1-x)-4000] 
será aplicado. Após 1 mês, Paulo terá [8000.(1-x)-4000]. 1,25 que deve ser 
igual à 2ª prestação. 
Então, [8000.(1-x)-4000].1,25 = 4000 = x=0,1 


Em 15 Feb 2004, [EMAIL PROTECTED] escreveu: 

Uma motocicleta cujo preço de 
tabela é R$ 8.000,00 é vendida , à vista , com desconto de x% ou 
em duas parcelas iguais 
 de R$ 4.000,00 , sendo a 
primeira no ato da compra e segunda um mês após a compra. Supondo 
que o Sr. Paulo dispõe 
 do dinheiro necessário 
para pagar à vista e que ele sabe que a diferença entre o preço à vista e 
a primeira parcela pode ser 
 aplicada no mercado financeiro a 
uma taxa de 25% ao mês. Nessas condições , qual é o valor de x que torna 
indiferente 
 comprar à vista ou a prazo 
: 
 
 a) 
10 
b)12 c) 
15 D) 16 
 
 -- 
 Esta mensagem foi verificada pelo sistema de antivírus e 
 acredita-se estar livre de perigo. 
 
-- 

_
Voce quer um iGMail protegido contra vírus e spams? 
Clique aqui: http://www.igmailseguro.ig.com.br



Re: [obm-l] Ajuda

2004-02-15 Por tôpico Fabio Henrique
Se você pretende estudar problemas mais profundos e demonstrativos, vale a 
pena o do Wagner. Outro que gosto é o do Timótheo Pereira, mas este vai ser 
difícil achar. 


Em 15 Feb 2004, [EMAIL PROTECTED] escreveu: 

Ficar bom em Geometria! 
 
- Original Message - 
From: Fabio Henrique 
To: 
Sent: Sunday, February 15, 2004 9:56 AM 
Subject: Re: [obm-l] Ajuda 
 
 Depende do teu propósito. 
 
 
 Em 14 Feb 2004, [EMAIL PROTECTED] escreveu: 
 
 Gostaria da ajuda dos amigos. 
  
  Qual destes dois livros de geometria plana é 
 melhor? 
  Os livros de geometria do Morgado e do Eduardo 
 Wagner(Geometria1 e 2) 
  
  Ou o Fundamentos da Matemática Volume 
 9 
  
 -- 
 
 _ 
 Voce quer um iGMail protegido contra vírus e spams? 
 Clique aqui: http://www.igmailseguro.ig.com.br 
 
 
= 
Instruções para entrar na lista, sair da lista e usar a lista em 
http://www.mat.puc-rio.br/~nicolau/olimp/obm-l.html 
= 
 
-- 

_
Voce quer um iGMail protegido contra vírus e spams? 
Clique aqui: http://www.igmailseguro.ig.com.br



Re: [obm-l] OI

2004-02-14 Por tôpico Fabio Henrique
Em 14 Feb 2004, [EMAIL PROTECTED] escreveu: 

OLA AMIGOS PODERIA AJUDAR NESTES 
QUESTÕES. 
 
1) Para se demarcar o 
estacionamento de todo o lado direito de uma rua reta, foram pintados 20 
retângulos de 4,5metros de comprimento e 2,5 metros de largura. Sabendo-se 
que 
os carros estacionam no sentido do comprimento dosretângulos e da rua, e à 
frente e atrás de cada um dos retângulos tem 50 centímetros de folga, qual 
é o 
comprimento, emmetros, da rua? 
 
A) 90 B) 
 90,5 C) 95 D) 100 E) 
 100,5 
 
2) Uma massa 
fermentada, ao ser colocada para descansar, ocupou uma área circular S de 
raio 
r. Após um certo tempo t,ela passou a ocupar uma área 21% maior que S. Qual 
o 
valor de r, em centímetros, para que a massa não transborde,quando colocada 
para 
descansar durante o tempo t, em um tabuleiro circular de raio 22 
centímetros? 
 

[[se a área aumenta 21% em um tempo t, neste mesmo tempo, o raio aumenta 
10%. Então a resposta é 20 cm, para que o raio aumente 10% e chegue a 22 
cm.]] 


3) Um bebedouro que usa 
garrafão de água tem 2,5 metros de serpentina por onde a água passa para 
gelar. 
Sabe-se quetal serpentina gasta 12 segundos para ficar totalmente gelada. 
Colocando-se um garrafão de 10 litros e ligando-se obebedouro, leva-se 5 
minutos 
para que toda a água saia gelada. Se nas mesmas condições, fosse colocado 
um 
garrafãode 20 litros no lugar do de 10 litros, o tempo gasto para que toda 
a 
água saísse gelada seria de: 
 
A) 9 minutos e 36 segundos. 
 
B) 9 minutos e 48 segundos. 
 
C) 10 minutos. 
 
D) 10 minutos e 12 segundos. 
 
E) 11 minutos. 
 
-- 
[[Dos 5 minutos, 12 segundos são para gelar a serpentina. Logo, para gelar o 
conteúdo do garrafão (10 litros), precisa-se de 4 min e 48 segundos = 4 + 
4/5 min = 24/5 min, sendo 24/50 min para cada litro. Para 20 litros temos 20 
x 24/50 min = 48/5 min = 9 + 3/5 min = 9 min 36 seg. Adicione os 12 seg da 
serpentina. Resp: B 

_
Voce quer um iGMail protegido contra vírus e spams? 
Clique aqui: http://www.igmailseguro.ig.com.br



Re: RES: RES: [obm-l] area de triangulo

2004-02-11 Por tôpico Fabio Henrique
Não faltou dizer que as circunferências são tangentes entre si duas a duas? 

Em 10 Feb 2004, [EMAIL PROTECTED] escreveu: 

![endif]-- 
 
 -- 
 
![endif]-- 
 
 Acho que é isso: 
 
 http ://www. klystron . kit . net /triangulo.jpg 
 
 -Mensagem original- 
 
 De: [EMAIL PROTECTED] puc -rio. br 
[mailto:[EMAIL PROTECTED] Em nome 
de Guilherme Carlos Moreira e Silva 
 
 Enviada em: terça-feira, 10 de 
fevereiro de 2004 19:52 
 
 Para: [EMAIL PROTECTED] puc -rio. br 
 
 Assunto: Re: RES: [obm-l] area de 
triangulo 
 
 bem 
ñ entendi bem o enunciado da questao e por isto ela me pareceu facil 
 
 poderia 
mandar uma figura? 
 
 Douglas Ribeiro Silva 
 com. br wrote: 
 
 Salvo engano sua área é 32[2sqrt(3) + 3] 
 
 Bom, o 
ângulo formado entre um lado do triangulo e um dos vértices do triangulo 
até o 
centro da circunferência mais próxima desse vértice é 30°. Desse centro até 
o 
lado são 4cm, pois ela é tangente. Como o ângulo é de 30° então do ponto de 
tangência até o vértice do triangulo vai ser 4sqrt(3) cm. Isso obviamente 
vale 
pro outro lado do triângulo. Logo pra descobrir o tamanho do lado falta só 
o 
meio do lado que é um segmento de 8cm, formado pela união dos 
centros das circunferências internas de raio 4cm. Logo o lado do triângulo 
vale 
4sqrt(3) + 8 + 4sqrt(3) = 8(sqrt (3 ) +1) cm. 
 
 Daí: 
 
 A= 
L²sqrt (3 )/4 
 
 Desenvolvendo 
dá 32[2sqrt(3) + 3] cm² 
 
 Avisem-me 
se por acaso saiu algo errado... Douglas Ribeiro 
 
 -Mensagem original- 
 
 De: [EMAIL PROTECTED] puc -rio. br [mailto:[EMAIL PROTECTED] 
 Em nome de [EMAIL PROTECTED] com 
 
 Enviada em: terça-feira, 10 de 
fevereiro de 2004 00:29 
 
 Para: [EMAIL PROTECTED] puc -rio. br 
 
 Assunto: [obm-l] area de triangulo 
 
 Ola pessoal, 
 
Imaginem um triangulo equilatero com 3 circunferencias 
de raio 4 cm inscritas neste triangulo. Cada lado do triangulo eh tangente 
a 2 circunferencia . Qual a area do triangulo 
? 
 
 Yahoo! 
Mail - 6MB, anti-spam e antivírus gratuito. Crie 
sua conta agora! 
 
-- 

_
Voce quer um iGMail protegido contra vírus e spams? 
Clique aqui: http://www.igmailseguro.ig.com.br



Re: [obm-l] RE: [obm-l] Iezzi dúvida

2004-01-28 Por tôpico Fabio Henrique
Achei mais fácil decompor o 5 em 10/2. 


Em 28 Jan 2004, [EMAIL PROTECTED] escreveu: 

O leonardo está correto, apenas temos que decompor 100 em 2^2 . 5^2 , ai 
fica fácil vc decompoe tb a parte do 2 e do 5 , depois disso somente é um 
jogo algebrico 
 
 até chegar em (2^2.5^2)^(x^2 - y) = (2^2.5^2) 
 
 Pérsio 
 
 Carlos Alberto wrote: 
 
 Como chegou nesse resultado?? 
 
 leonardo mattos wrote: 
 Ola, 
 
Antes de substituir desenvolva a equacao (I) e vc vera que (x^2-y)=1 
 
Um abraço, 
Leonardo 
 
From: Tâni Aparecida 
Reply-To: [EMAIL PROTECTED] 
To: [EMAIL PROTECTED] 
Subject: [obm-l] Iezzi dúvida 
Date: Wed, 28 Jan 2004 12:31:03 -0300 (ART) 
 
Resolva o seguinte sistema: 
 
2^(2.(x^2-y)) = 100 . 5^(2.(y-x^2)) ( I ) 
x + y = 5 ( II ) 
 
 
---
 
 
Comecei a resolver dessa maneira em ( II ) tenho que 
y = 5-x substituo em ( I ) que fica 
 
2^(2.(x^2+x-5)) = 100 . 5^(2.(-x^2 -x + 5)) 
 
e agora como faço para resolver isso? 
 
 
 
- 
Yahoo! Mail - 6! 
! MB, 
 anti-spam e antivírus gratuito. Crie sua conta agora! 
 
_ 
MSN Hotmail, o maior webmail do Brasil. http://www.hotmail.com 
 
= 
Instruções para entrar na lista, sair da lista e usar a lista em 
http://www.mat.puc-rio.br/~nicolau/olimp/obm-l.html 
= 
 
-- 

_
Voce quer um iGMail protegido contra vírus e spams? 
Clique aqui: http://www.igmailseguro.ig.com.br



Re: [obm-l] Impossibilidade do movimento

2004-01-24 Por tôpico Fabio Henrique
Acho curioso que sempre que se toca no assunto Paradoxo de Aquiles e a 
Tartaruga, de Zenon, sempre se recorre a somas infinitas como explicação do 
paradoxo. Mesmo quando o assunto foi questão da prova da UFRJ, o argumento 
usado foi o mesmo. 
Parece-me que a explicação do paradoxo é o fato de que este foi construído 
sobre condições idealizadas e não reais. Há um momento em que a distância 
entre Aquiles e a tartaruga seria tão pequena (segundo as parcelas da soma 
infinita) que chegaria a ser menor do que o pé da tartaruga. Nunca 
vi/ouvi/li ninguém argumentar que o paradoxo criado por Zenon considera 
tanto a tartaruga quanto Aquiles como objetos pontuais, sem dimensão. O que 
de fato contraria o nosso senso prático. 
Estaria eu pensando bobagem? 

24 Jan 2004, [EMAIL PROTECTED] escreveu: 

Partindo desse princípio, pode-se dizer que a cada termo adicionado naquela 
soma, o valor total aumenta. Por exemplo, se eu utilizar 10 termos eu tenho 
um valor; se eu utilizar 100 termos eu tenho outro maior, e assim 
sucessivamente. Desse modo, como a soma é infinita e possui estritamente 
termos positivos, seu resultado deveria ser infinito. No entanto, pelos 
conhecimentos atuais de matemática, isso não ocorre. Muito estranho! 
 
- Original Message - 
From: Frederico Reis Marques de Brito 
To: 
Sent: Saturday, January 24, 2004 9:47 AM 
Subject: Re: [obm-l] Impossibilidade do movimento 
 
 Isto é absolutamente falso. Observe que 1/(10^n) tende a 0 quando 
n 
 tender a infinito, de forma estritamente decrescente, isto é , se n  m 
= 
 1/(10^n)  1/(10^m), mas 0 não é um termo dessa sequência. Posto isto , é 
 fácil ver que não existe um menor número e que as demais parcelas são 
 múltiplas desta... 
 
 Frederico. 
 
 
 From: Marcelo Augusto Pereira 
 Reply-To: [EMAIL PROTECTED] 
 To: 
 Subject: Re: [obm-l] Impossibilidade do movimento 
 Date: Fri, 23 Jan 2004 22:10:01 -0200 
  
 O fato de essa soma ser calculável(1/9) não indica que existe um número 
de 
 valor muito pequeno e que esse número seria o valor mínimo que possa 
 existir? Assim todos os outros números seriam múltiplos desse menor 
valor 
 possível, ou seja, esse número seria algo como um valor quântico. Dessa 
 forma, também existiria uma unidade quântica de deslocamento linear, o 
que 
 faria com que a quantidade de pontos em um segmento de reta não fosse 
 infinita e o movimento fosse possível. Se para cada número existisse um 
 menor, a soma teria que ser infinita, e o resultado infinito. 
  
 - Original Message - 
 From: Frederico Reis Marques de Brito 
 To: 
 Sent: Friday, January 23, 2004 9:27 PM 
 Subject: RE: [obm-l] Impossibilidade do movimento 
  
  
   
   Essencialmente esse problema é ujm dos paradoxos de Zenão, um grego 
 antigo 
   que usava a idéia de infinito para chegar a conclusões aparentemente 
   absurdas, tais como a impossibilidade do movimento, por exemplo. 
Agora 
 vou 
   dar uma de Dirichlet, o da lista é claro: Pense no seguinte, uma soma 
de 
   infinitas parcelas positivas é sempre infinito, ou não 
necessariamente? 
 Para 
   ajudar nessa resposta, pense em calcular, por exemplo: 1/10 + 1/100 + 
 1/1000 
   + ... . Bom e agora, o que tudo isto tem a ver com sua pergunta? 
   
   Espero ter ajudado, apesar dessa resposta meio enigmática, mas acho 
que 
   assim auxilio mais! 
   
   Frederico. 
   
   From: Marcelo Augusto Pereira 
   Reply-To: [EMAIL PROTECTED] 
   To: 
   Subject: [obm-l] Impossibilidade do movimento 
   Date: Fri, 23 Jan 2004 19:05:25 -0200 

   Entre dois números reais há infinitos outros. Considere um segmento 
de 
 reta 
   com o número 0 assinalado em uma ponta e o número 1 marcado na 
outra. 
   Considere também que esse segmento de reta foi representado no chão 
com 
 um 
   risco de um metro de comprimento. Para cada número entre 0 e 1 há um 
 ponto 
   correspondente no segmento de reta e, conseqüentemente, no risco 
 marcado 
 no 
   chão. Como eu consigo caminhar do ponto 0 até o ponto 1, se para 
chegar 
 de 
   0 
   até 1 eu tenho que passar por infinitos pontos? 

   
  
= 
   Instruções para entrar na lista, sair da lista e usar a lista em 
   http://www.mat.puc-rio.br/~nicolau/olimp/obm-l.html 
   
  
= 
   
   _ 
   MSN Hotmail, o maior webmail do Brasil. http://www.hotmail.com 
   
   
 
= 
   Instruções para entrar na lista, sair da lista e usar a lista em 
   http://www.mat.puc-rio.br/~nicolau/olimp/obm-l.html 
   
 
= 
   
  
 
= 
 Instruções para entrar na lista, sair da lista e usar a lista em 
 

Re: [obm-l] CN-97_

2004-01-21 Por tôpico Fabio Henrique
Se escrever 147 algarismos você terá 31707 vinte e nove vezes e, a seguir, 3 
e 1 que somam 526. Para exceder de 530 é necessário o próximo dígito: 7. 

Resp: E 

Em 20 Jan 2004, [EMAIL PROTECTED] escreveu: 

 Um aluno, efetuando a divisão de 13 por 41, foi 
 determinando o quociente até a soma de todos os 
 algarismos por ele escritos, na parte decimal, foi 
 imediatamente maior ou igual a 530. Quantas casas 
 decimais ele escreveu? 
 
 (A) 144 (D) 147 
 (B) 145 (E) 148 
 (C) 146 
 
13/41 = 0.31707317073... 
 
É uma dízima períodica. 
 
Temos que 3 + 1 + 7 + 0 + 7 = 18 
Então temos 530/18 = 29.44... 
 
Então, 5*29.4... = 147.22 
 
Achei letra (D). 
 
Abraço, 
Henrique. 
 
= 
Instruções para entrar na lista, sair da lista e usar a lista em 
http://www.mat.puc-rio.br/~nicolau/olimp/obm-l.html 
= 
 
-- 

_
Voce quer um iGMail protegido contra vírus e spams? 
Clique aqui: http://www.igmailseguro.ig.com.br



Re: [obm-l] OBM:LISTA MONTE DE MERDA

2004-01-04 Por tôpico Fabio Henrique
Olha, eu realmente gostaria de saber o motivo de tanta raiva. Eu sei que 
talvez jamais saiba porque insiste em provocar os participantes desta lista, 
porque este ódio para com o Nicolau e porque perde tempo com isso. 
Por que manter este comportamento psicótico e previsível. Agora, você vai 
dirigir os seus xingamentos a mim. Tudo isso faz parte deste comportamento 
típico de pessoas que não conseguem lidar com dificuldades mantendo o 
equilíbrio. 
Cara, já deu pra ver que você é bom pra cacete quando se trata de 
computadores. Se eu soubesse metade do que você sabe sobre o assunto, 
certamente seria o fundador e moderador de uma lista sobre isto. Mostraria a 
parte prática do conhecimento. 
Talvez, você ainda seja egoísta o bastante para tomar este tipo de 
iniciativa. 
E agora? Você vai me bombardear? É possível e muito provável. No entanto, se 
você pelo menos parar para considerar o que escrevo, numa boa, sem este 
desdém costumeiro, quem sabe muitas pessoas consigam crescer graças a um 
movimento seu, quem sabe, daqui a alguns anos, muitos lhe sejam gratos. 
Fabio. 


Em  1 Jan 2004, [EMAIL PROTECTED] escreveu: 

MagazinesNewspapersNewslettersAll Products 
More Search Options 
 
Professional  Trade 
Education 
Engineering 
Medicine 
 See all 
 
- 
--- 
Browse by Subject 
Arts  Crafts 
Automotive 
Bridal 
Business  Finance 
Children's 
Computer  Internet 
Electronics  Audio 
Entertainment 
Family  Parenting 
Fashion  Style 
Food  Gourmet 
Games  Hobbies 
Gay  Lesbian 
Health  Fitness 
History 
Home  Garden 
International 
Lifestyle  Cultures 
Literary 
Men's Interest 
Music 
News  Politics 
Newspapers 
Pets 
Professional  Trade 
Religion  Spirituality 
Science  Nature 
Spanish-Language 
Sports  Leisure 
Teens 
Travel  Regional 
Women's Interest 
 
Need Help? 
 
Magazine Subscriptions FAQ 
Help Desk 
Publisher Services 
Send Feedback 
 
Health  Fitness Magazines 
 Inspire your mind and body--choose from over 200 
Health  Fitness magazines, including Men's Health, 
Shape, and Self. 
 
Subscribe and Save 
 Get $5 to $20 off a future Amazon.com purchase when 
you subscribe to one of our select magazine titles-- 
including People, Wired, and Sports Illustrated. 
 
Classic Magazines 
 If you're a devoted subscriber to National Geographic, 
we recommend trying one of these other quality 
magazines: National Geographic Kids, National Geographic 
Traveler, or National Geographic Adventure. 
 
Movers and Shakers 
Scientific American 
 766% today ~ Amazon.com Sales Rank: 6 (was 52) 
From Amazon.com 
For working scientists, especially in high-tech fields, 
there are only a few crucial nonjournal periodicals to 
pore over faithfully, and Scientific American is one 
of... Read more 
 
 800% Real Simple [MAGAZINE SUBSCRIPTION] Magazine 
Subscription 
 787% Southern Living [MAGAZINE SUBSCRIPTION] Magazine 
Subscription 
 783% Hot Rod [MAGAZINE SUBSCRIPTION] Magazine 
Subscription 
 
 More Movers  Shakers 
 
Your Lifestyle 
Water Sports: Make Waves 
 
Boating Life [MAGAZINE SUBSCRIPTION] 
Magazine Subscription 
Alkaline AA 12-Pack Batteries 
by Rayovac 
 
More in Water Sports: Make Waves 
 Deal of the Day 
 Subscribe to the new music magazine Tracks and get $15 
off a future Amazon.com purchase. 
 
Subscriptions for $10 or Less 
 Stuff their stocking with a magazine subscription for 
$10 or less, including Good Housekeeping, Esquire, and 
Parents. 
 
Top Selling Subscriptions 
Our Bestsellers, Updated Hourly 
 
InStyle [MAGAZINE SUBSCRIPTION] Magazine Subscription 
O : The Oprah Magazine [MAGAZINE SUBSCRIPTION] Magazine 
Subscription 
Real Simple [MAGAZINE SUBSCRIPTION] Magazine 
Subscription 
Sports Illustrated [MAGAZINE SUBSCRIPTION] Magazine 
Subscription 
Smithsonian [MAGAZINE SUBSCRIPTION] Magazine 
Subscription 
Scientific American [MAGAZINE SUBSCRIPTION] Magazine 
Subscription 
Wired [MAGAZINE SUBSCRIPTION] Magazine Subscription 
ESPN the Magazine [MAGAZINE SUBSCRIPTION] Magazine 
Subscription 
 More Top Sellers 
 
Top Selling Subscriptions in Business  Finance 
SmartMoney [MAGAZINE SUBSCRIPTION] Magazine Subscription 
$12.00 
Consumer Reports [MAGAZINE SUBSCRIPTION] Magazine 
Subscription $26.00 
The Economist [MAGAZINE SUBSCRIPTION] Magazine 
Subscription $129.00 
 More Top Selling Subscriptions in Business  Finance 
 
- 
--- 
More to Explore 
E-mail Recommendations 
Learn about Co-op Build Your Own Online Magazine Store 
 Team up with Amazon.com to sell magazines from your Web 
site. We'll help you. Join Amazon.com Associates today! 
Sell Your Used Items 
 
__ 
Acabe com aquelas janelinhas que pulam na sua tela. 
AntiPop-up UOL - É grátis! 
http://antipopup.uol.com.br/ 
 
= 
Instruções para entrar na lista, sair da 

Re: [obm-l] erro no mathematica?

2003-12-10 Por tôpico Fabio Henrique
Como consigo o mathematica? 


Em 09 Dec 2003, [EMAIL PROTECTED] escreveu: 

pessoal, pq no mathematica (5.0) quando eu calculo 
 
Integral de 0 até pi de Abs[1/2 + Cos[t]] dt 
ele me devolve -pi/2 ? 
a resposta nao é sqrt(3) + pi/6 ? 
 
obrigado! 
 
= 
Instruções para entrar na lista, sair da lista e usar a lista em 
http://www.mat.puc-rio.br/~nicolau/olimp/obm-l.html 
= 
 
-- 

_
Voce quer um iGMail protegido contra vírus e spams? 
Clique aqui: http://www.igmailseguro.ig.com.br
Ofertas imperdíveis! Link: http://www.americanas.com.br/ig/
Ofertas imperdíveis!

=
Instruções para entrar na lista, sair da lista e usar a lista em
http://www.mat.puc-rio.br/~nicolau/olimp/obm-l.html
=


Re: [obm-l] erro no mathematica?

2003-12-10 Por tôpico Fabio Henrique
Obrigado pelo esclarecimento mas o comentário me ofendeu.Em 10 Dec 2003, 
[EMAIL PROTECTED] escreveu: 

On Wed, Dec 10, 2003 at 06:53:36AM -0200, Fabio Henrique wrote: 
 Como consigo o mathematica? 
 
É um software comercial, para tê-lo legalmente você precisa comprar. 
A home page do produto é 
http://www.wolfram.com/products/mathematica/index.html 
 
E por favor, nada de discutir pirataria aqui. 
 
[]s, N. 
= 
Instruções para entrar na lista, sair da lista e usar a lista em 
http://www.mat.puc-rio.br/~nicolau/olimp/obm-l.html 
= 
 
-- 

_
Voce quer um iGMail protegido contra vírus e spams? 
Clique aqui: http://www.igmailseguro.ig.com.br
Ofertas imperdíveis! Link: http://www.americanas.com.br/ig/
Ofertas imperdíveis!

=
Instruções para entrar na lista, sair da lista e usar a lista em
http://www.mat.puc-rio.br/~nicolau/olimp/obm-l.html
=


Re: [obm-l] erro no mathematica?

2003-12-10 Por tôpico Fabio Henrique
Alguém pode me esclarecer o que é este mathematica? Onde consigo. Desculpem 
a ignorância... 


Em 10 Dec 2003, [EMAIL PROTECTED] escreveu: 

-- 
 
 No mathematica 4, a resposta é: 
 
 -pi/2 +( 3Sqrt[3] + 2pi)/3 = Sqrt[3] + pi/6 
 
 ---Original Message--- 
 
 From: [EMAIL PROTECTED] 
 Date: quarta-feira, 10 de dezembro de 2003 10:35:38 
 To: [EMAIL PROTECTED] 
 Subject: Re: [obm-l] erro no mathematica? 
 
  Integral de 0 até pi de Abs[1/2 + Cos[t]] dt 
  ele me devolve -pi/2 ? 
 
 a integral de uma função f(x)= 0 no intervalo de 
 integração NUNCA será um número negativo. Certamente é 
 um erro do Mathematica. 
 
 __ 
 Do you Yahoo!? 
 New Yahoo! Photos - easier uploading and sharing. 
 http://photos.yahoo.com/ 
 = 
 Instruções para entrar na lista, sair da lista e usar a lista em 
 http://www.mat.puc-rio.br/~nicolau/olimp/obm-l.html 
 = 
 . 
 
  
 IncrediMail - Email has finally evolved - 
 
-- 

_
Voce quer um iGMail protegido contra vírus e spams?
Clique aqui: http://www.igmailseguro.ig.com.br
Ofertas imperdíveis! Link: http://www.americanas.com.br/ig/
Ofertas imperdíveis!

=
Instruções para entrar na lista, sair da lista e usar a lista em
http://www.mat.puc-rio.br/~nicolau/olimp/obm-l.html
=


Re: [obm-l] erro no mathematica?

2003-12-10 Por tôpico Fabio Henrique
Alguém pode me esclarecer o que é este mathematica? Onde consigo. Desculpem 
a ignorância... 


Em 10 Dec 2003, [EMAIL PROTECTED] escreveu: 

-- 
 
 No mathematica 4, a resposta é: 
 
 -pi/2 +( 3Sqrt[3] + 2pi)/3 = Sqrt[3] + pi/6 
 
 ---Original Message--- 
 
 From: [EMAIL PROTECTED] 
 Date: quarta-feira, 10 de dezembro de 2003 10:35:38 
 To: [EMAIL PROTECTED] 
 Subject: Re: [obm-l] erro no mathematica? 
 
  Integral de 0 até pi de Abs[1/2 + Cos[t]] dt 
  ele me devolve -pi/2 ? 
 
 a integral de uma função f(x)= 0 no intervalo de 
 integração NUNCA será um número negativo. Certamente é 
 um erro do Mathematica. 
 
 __ 
 Do you Yahoo!? 
 New Yahoo! Photos - easier uploading and sharing. 
 http://photos.yahoo.com/ 
 = 
 Instruções para entrar na lista, sair da lista e usar a lista em 
 http://www.mat.puc-rio.br/~nicolau/olimp/obm-l.html 
 = 
 . 
 
  
 IncrediMail - Email has finally evolved - 
 
-- 

_
Voce quer um iGMail protegido contra vírus e spams?
Clique aqui: http://www.igmailseguro.ig.com.br
Ofertas imperdíveis! Link: http://www.americanas.com.br/ig/
Ofertas imperdíveis!

=
Instruções para entrar na lista, sair da lista e usar a lista em
http://www.mat.puc-rio.br/~nicolau/olimp/obm-l.html
=


[obm-l] Re: Equação

2003-12-04 Por tôpico Fabio Henrique
Pedro, observe que o primeiro módulo zera para x = 1 e o segundo para x = 
-5. Vamos então dividir a nossa reta real em 3 pedaços: MENOR QUE OU IGUAL A 
-5, ENTRE -5 E 1 e MAIOR QUE OU IGUAL A 1. 

Na primeira região, trocaremos mod(x-1) por -x+1  e  mod(x+5) por -x-5. 
Assim ficaremos com: 

-2x-4=6 = x=-5. Este resultado está no intervalo em que estamos 
trabalhando, portanto é válido. 

Na segunda região, trocaremos mod(x-1) por -x+1 e mod(x+5) por x+5. Assim 
ficaremos com: 

x+5-x+1=6 = 6=6. Isto quer dizer que todos os valores deste intervalo são 
soluções da tua equação. 

Na terceira região, trocaremos mod(x-1) por x-1 e mod(x+5) por x+5. Assim 
ficaremos com: 

2x+4=6 = x=1. Este resultado está no intervalo em que estamos trabalhando, 
portanto é válido. 

Assim, a resposta é [-5,1] 



Em 4 Dec 2003, [EMAIL PROTECTED] escreveu: 

oi, pessoal 
 
 Não conseguir 
resolver estas questões de um concurso , ajude-me 
 
 1) A soma dos valores inteiros positivos e 
negativos de x que satisfazem a equação: 
 
l x - 1 l + l x + 5 l = 6 
 ( equação modular ) 
 
 a) 
-14 
 
b) -13 
 c) -12 d) 
-11 
 
 2) Usando uma vez a letra X , uma vez a letra 
Y e N - 2 vezes a letra Z , podemos formar 20 anagramas diferentes 
com n letras em cada anagrama. O valor de n^2 é : 
 
 a) 
9 
b)16 
c)25 d) 36 
 
 3) ma motocicleta cujo preço de tabela é R$ 
8.000,00 é vendida , à vista , com desconto de x% ou em duas 
parcelas iguais 
 de R$ 4.000,00 , sendo a 
primeira no ato da compra e segunda um mês após a compra. Supondo 
que o Sr. Paulo dispõe 
 do dinheiro necessário 
para pagar à vista e que ele sabe que a diferença entre o preço à vista e 
a primeira parcela pode ser 
 aplicada no mercado financeiro a 
uma taxa de 25% ao mês. Nessas condições , qual é o valor de x que torna 
indiferente 
 comprar à vista ou a prazo 
: 
 
 a) 
10 
b)12 c) 
15 D) 16 
 
 -- 
 Esta mensagem foi verificada pelo sistema de antivírus e 
 acredita-se estar livre de perigo. 
 
-- 

_
Voce quer um iGMail protegido contra vírus e spams? 
Clique aqui: http://www.igmailseguro.ig.com.br
Ofertas imperdíveis! Link: http://www.americanas.com.br/ig/
Ofertas imperdíveis!

=
Instruções para entrar na lista, sair da lista e usar a lista em
http://www.mat.puc-rio.br/~nicolau/olimp/obm-l.html
=


Re: [obm-l] a^3+b^3+c^3 = 3abc

2003-11-09 Por tôpico Fabio Henrique
Pessoal, consegui fazendo (a+b+c)^3. 
Isto vai dar a^3 + b^3 + c^3 + 3ba^2 + 3ab^2 + 3ca^2 + 3ac^2 + 3bc^2 + 3cb^2 
+ 6abc = 0. 

a^3 + b^3 + c^3 + 3ab(a+b) + 3ac(a+c) + 3cb(b+c) + 6abc = 0. 

a^3 + b^3 + c^3 + 3ab(-c) + 3ac(-b) + 3cb(-a) + 6abc = 0. 

Abraços. Fabio Henrique. 



Gostaria de tentar uma resoluçao sobre o enunciado, só que fazendo um 
caminho inverso: 
 
Dado a+b+c=0, 
quero chegar em 
 
a^3 + b^3 + c^3 - 3abc = 0. 
 
Partindo de: 
 
a^3 + b^3 + c^3 - 3abc 
 
Farei a linha acima por determinante: 
 
 a b c 
 c a b 
 b c a 
 
A soma de cada linha deste deteminante eh a+b+c que como jah eh sabido eh 
zero. 
 
logo o determinante acima eh igual a zero. 
 
Assim temos: 
 
a^3 + b^3 + c^3 - 3abc = 0 
 
e 
 
a^3 + b^3 + c^3 = 3abc 
 
Por favor me corrijam se eu estiver errado. 
 
Obrigado. 
 
-- Mensagem original -- 
 
 Ola pessoal, 
  
 Depois de alguns meses afastado da lista e sem estudar matematica, pois 
 
 estava estudando para um concurso e acabei de faze-lo. Agora eh esperar 
ansioso 
  
 pelo resultado que sairah em menos de 2 semanas. Para nao ficar 
off-topic 
 vou 
 re-comecar a postar minhas duvidas. Vamos la: 
  
 1) Prove que se a + b + c = 0, entao a^3 + b^3 + c^3 = 3abc 
  
 Obs: Como estou voltando agora, desculpem me se o problema for trivial. 
 
 Preciso me desenferrujar aos poucos ;-) em matematica e pegar o ritmo de 
 novo. 
  
  
  
 
 
 
-- 
Use o melhor sistema de busca da Internet 
Radar UOL - http://www.radaruol.com.br 
 
 
 
Instruções 
para entrar na lista, sair da lista e usar a lista em 
http://www.mat.puc-rio.br/~nicolau/olimp/obm-l.html 
 
_ 
MSN Hotmail, o maior webmail do Brasil. http://www.hotmail.com 
 
Instruções 
para entrar na lista, sair da lista e usar a lista em 
http://www.mat.puc-rio.br/~nicolau/olimp/obm-l.html 
 
 
-- 

_
Voce quer um iGMail protegido contra vírus e spams? 
Clique aqui: http://www.igmailseguro.ig.com.br
Ofertas imperdíveis! Link: http://www.americanas.com.br/ig/
Ofertas imperdíveis!

=
Instruções para entrar na lista, sair da lista e usar a lista em
http://www.mat.puc-rio.br/~nicolau/olimp/obm-l.html
=


Re: [obm-l] equação!!

2003-11-09 Por tôpico Fabio Henrique
Observe que f(4)=-387 e f(5)=53. Então existe uma raiz entre 4 e 5. Sugiro 
utilizar o método de Newton-Raphson 
Abraços. Fabio Henrique. 

Em 28 Oct 2003, [EMAIL PROTECTED] escreveu: 

como posso resolver a seguinte equação? 
 
 (x^4) + (x^3) + (x^2) + (x) = 727 
 
 (consegui fazer na máquina, pois ainda naum tive muito tempo para 
resolvê-la algebricamente). 
 
-- 

_
Voce quer um iGMail protegido contra vírus e spams? 
Clique aqui: http://www.igmailseguro.ig.com.br
Ofertas imperdíveis! Link: http://www.americanas.com.br/ig/
Ofertas imperdíveis!

=
Instruções para entrar na lista, sair da lista e usar a lista em
http://www.mat.puc-rio.br/~nicolau/olimp/obm-l.html
=


Re: [obm-l] Re: N/A correçao

2003-11-08 Por tôpico Fabio Henrique
Pessoal, resolvi usando o 1º lema de Kaplansky. Desta forma, calculei 
quantos são sem zeros, com exatamente 1 zero, com exatamente 2 zeros, com 
exatamente 3 zeros e com exatamente 4 zeros. Encontrei, respectivamente, os 
valores 1, 8, 21, 20 e 5. Somados dão 55. 

Abraços, Fabio Henrique. 



 Em 4 Nov 2003, [EMAIL PROTECTED] escreveu: 

Conforme o Stabel ja apontou, ha um erro de soma abaixo. 
f(7) = 21 + 13 = 34 e f(8) = 34+21 = 55. 
 
-- Original Message --- 
From: Augusto Cesar de Oliveira Morgado 
To: [EMAIL PROTECTED] 
Sent: Mon, 3 Nov 2003 20:59:29 -0200 
Subject: [obm-l] Re: N/A 
 
 Seja f(n) a resposta para uma sequencia de n bits. Ou a seq. começa 
 em 1 ou começa em 01. Logo, f(n)=f(n-1)+f(n-2). Como f(1) = 2 e f(2) 
 = 3, f(3) = 2+3=5, f(4) = 5+3 = 8, f(5) = 8+5 = 13, f(6)=8 = 21, 
 f(7) = 21+13 = 44 e f(8) = 44+21 = 65. 
 
 -- 
 CIP WebMAIL - Nova Geração - v. 2.1 
 CentroIn Internet Provider http://www.centroin.com.br 
 Tel: (21) 2542-4849, (21) 2295-3331 Fax: (21) 2295-2978 
 Empresa 100% Brasileira - Desde 1992 
 
 -- Original Message --- 
 From: Daniel Faria 
 To: [EMAIL PROTECTED] 
 Sent: Mon, 03 Nov 2003 19:16:55 -0200 
 Subject: N/A 
 
  Ainda nao consegui finalizar este exercício: 
  
  De quantas maneiras podemos formar uma sequencia de oito bits(0 ou 1) 
  de forma que nunca apareça nesta sequencia zeros adjacentes ( _ _ 
  0 0 _ _ _ _ ). 
  
  Obrigado. 
  
  _ 
  MSN Hotmail, o maior webmail do Brasil. http://www.hotmail.com 
  
  
  
Instruções para entrar na lista, sair da lista e usar a lista em 
  http://www.mat.puc-rio.br/~nicolau/olimp/obm-l.html 
  
 
--- End of Original Message --- 
 
  
Instruções para entrar na lista, sair da lista e usar a lista em 
 http://www.mat.puc-rio.br/~nicolau/olimp/obm-l.html 
 
--- 
End of Original Message --- 
 
Instruções 
para entrar na lista, sair da lista e usar a lista em 
http://www.mat.puc-rio.br/~nicolau/olimp/obm-l.html 
 
 
-- 

_
Voce quer um iGMail protegido contra vírus e spams? 
Clique aqui: http://www.igmailseguro.ig.com.br
Ofertas imperdíveis! Link: http://www.americanas.com.br/ig/
Ofertas imperdíveis!

=
Instruções para entrar na lista, sair da lista e usar a lista em
http://www.mat.puc-rio.br/~nicolau/olimp/obm-l.html
=


Re: [obm-l]

2003-10-17 Por tôpico Fabio Henrique
Nicolau, sou só eu que estou recebendo 50 mensagens iguais a esta por dia? o 
que está havendo? 

Abraços. Fabio. 


Em 16 Oct 2003, [EMAIL PROTECTED] escreveu: 

= 
Instruções para entrar na lista, sair da lista e usar a lista em 
http://www.mat.puc-rio.br/~nicolau/olimp/obm-l.html 
= 
 
-- 

_
Voce quer um iGMail protegido contra vírus e spams?
Clique aqui: http://www.igmailseguro.ig.com.br
Ofertas imperdíveis! Link: http://www.americanas.com.br/ig/
Ofertas imperdíveis!

=
Instruções para entrar na lista, sair da lista e usar a lista em
http://www.mat.puc-rio.br/~nicolau/olimp/obm-l.html
=


Re: [obm-l] Triângulos_(Mr._Crowley)

2003-09-11 Por tôpico Fabio Henrique
Estava lendo mensagens antigas e acho que encontrei um erro. O lado AB vale 
2sqrt(13). 
Fabio. 





Em 4 Sep 2003, [EMAIL PROTECTED] escreveu: 

Legal cara,ce e o mesmo que foi homenageado pelo 
Ozzy Osbourne ou colocou este nick como eu fiz o 
meu? 
 --- Andre Araujo escreveu: 
 
  
 AB é a hipotenusa de um triângulo retângulo 
 ABC. A 
 mediana AD mede 7 e a mediana BE mede 4. O 
 comprimento 
 AB é igual a: 
 
 Pitagoras no triangulo BCE: BC^2+(AC/2)^2=BE^2 
 Pitagoras no triangulo ACD: (BC/2)^2+AC^2=AD^2 
 
 Somando as duas equacoes, temos: 
 
 (5/4)*(BC^2+AC^2)=16+49, mas BC^2+AC^2=AB^2. 
 Logo: 
 
 AB^2=4*65/5=AB=2sqrt(15). 
 
  
 a)2·sqrt(3) 
 b)5·sqrt(2) 
 c)5·sqrt(3) 
 d)10 
 e)n.d.a 
  
 ABC é um triângulo e M é um ponto médio sobre 
 o lado 
 BC, tal que MC=2MB. A razão entre as área dos 
 triângulos 
 ABC e MAC é: 
 
 Note que a altura relativa ao lado BC(h)do 
 triangulo ABC eh igual a altura 
 relativa ao lado MC do triangulo AMC. Logo: 
 
 S(ABC)=BC*h/2 
 S(AMC)=MC*h/2 
 
 S(ABC)/S(AMC)=BC/MC=(MC+MB)/MC=(2*MB+MB)/2*MB = 
 3/2. 
 
  
 a)4 b)3 c)2 d)9/4 e)3/2 
  
 Grato 
  
 Mr. Crowley 
  
 
__ 
 
 Acabe com aquelas janelinhas que pulam na sua 
 tela. 
 AntiPop-up UOL - É grátis! 
 http://antipopup.uol.com.br/ 
  
 
= 
 
 Instruções para entrar na lista, sair da lista 
 e usar a lista em 
 
http://www.mat.puc-rio.br/~nicolau/olimp/obm-l.html 
 
 
= 
 
  
 -- 
 
 
_ 
 Voce quer um iGMail protegido contra vírus e 
 spams? 
 Clique aqui: http://www.igmailseguro.ig.com.br 
 Ofertas imperdíveis! Link: 
 http://www.americanas.com.br/ig/ 
 
 
= 
 Instruções para entrar na lista, sair da lista 
 e usar a lista em 
 
http://www.mat.puc-rio.br/~nicolau/olimp/obm-l.html 
 
= 
 
___ 
Desafio AntiZona: participe do jogo de perguntas e respostas que vai 
dar um Renault Clio, computadores, câmeras digitais, videogames e muito 
mais! www.cade.com.br/antizona 
= 
Instruções para entrar na lista, sair da lista e usar a lista em 
http://www.mat.puc-rio.br/~nicolau/olimp/obm-l.html 
= 
 
-- 

_
Voce quer um iGMail protegido contra vírus e spams? 
Clique aqui: http://www.igmailseguro.ig.com.br
Ofertas imperdíveis! Link: http://www.americanas.com.br/ig/
Ofertas imperdíveis!

=
Instruções para entrar na lista, sair da lista e usar a lista em
http://www.mat.puc-rio.br/~nicolau/olimp/obm-l.html
=


Re: [obm-l] poderia dar uma ajudinha por favor!!!!

2003-08-31 Por tôpico Fabio Henrique
23)vou calcular, em 1º lugar, de quantas formas podemos colocar os 6 de modo 
que apenas 1 casal fique junto: 
I. escolher o casal que terá o privilégio: 3 
II. permuta entre tal homem e seu par: 2 
III. escolher vizinho de tal homem (que não seja seu par):4 
IV. escolher o vizinho deste vizinho:2 
V. escolher o seguinte:1 
VI. último:1 
TOTAL = 3x2x4x2x1x1 = 48 

Agora, com exatamente 2 casais juntos: 
I. escolher os dois casais: C3,2 = 3 
II. permuta no 1º casal = 2 
III. permuta no 2º casal = 2 
Para que esta configuração seja possível, é necessário que os integrantes do 
terceiro casal fiquem, cada um de um lado, entre os dois casais escolhidos 
em I. 
IV. colocação destes dois últimos = 2 
TOTAL = 3x2x2x2 = 24 

Por fim, com os 3 casais juntos. 
I.permuta no 1º casal = 2 
II. permuta no 2º casal = 2 
III.permuta no 3º casal = 2 
IV. Permuta entre os casais = 2 
TOTAL = 2x2x2x2 = 16 

48 + 24 + 16 = 88 

Resposta = (6!/6) - 88 = 32 





Em 30 Aug 2003, [EMAIL PROTECTED] escreveu: 

23 -De quantos modos 3 casais podem sentar-se ao redor de uma mesa circular 
de tal forma que marido e mulher não fiquem juntos? 
 
R=32 
 
24 - Considere as proposições: 
 
I)Sendo A um conjunto com 3 elementos e B um conjunto com 5 elementos, 
existem 125 funções injetoras de A em B. 
II)O número de anagramas da palavra CLARA em que as letras AR aparecem 
juntas e nesta ordem é igual a 48. 
III)De um baralho de 12 cartas, das quais 4 são ases distintos (A-A-A-A), 
retiram-se 3 cartas ao acaso. A probabilidade de haver pelo menos um ás (A) 
entre as cartas retiradas é de . 
 
Analisando cada uma delas, pode-se afirmar que 
 
a)todas são verdadeiras. c)apenas I é falsa. 
b)todas são falsas. d)apenas III é verdadeira. 
 
30 - Dada uma cunha esférica de diedro 45° e raio 4 cm, tem-se que o volume 
da cunha e a área de sua superfície são, respectivamente ? 
 
_ 
Voce quer um iGMail protegido contra vírus e spams? 
Clique aqui: http://www.igmailseguro.ig.com.br 
Ofertas imperdíveis! Link: http://www.americanas.com.br/ig/ 
 
= 
Instruções para entrar na lista, sair da lista e usar a lista em 
http://www.mat.puc-rio.br/~nicolau/olimp/obm-l.html 
= 
 
-- 

_
Voce quer um iGMail protegido contra vírus e spams? 
Clique aqui: http://www.igmailseguro.ig.com.br
Ofertas imperdíveis! Link: http://www.americanas.com.br/ig/
Ofertas imperdíveis!

=
Instruções para entrar na lista, sair da lista e usar a lista em
http://www.mat.puc-rio.br/~nicolau/olimp/obm-l.html
=


Re: [obm-l] CN - 2001_AJUDEM-ME

2003-08-31 Por tôpico Fabio Henrique
Se o enunciado está corretamente transcrito, você já sabe que z = 1. 
Daí fica fácil... 


Encontrei x=90 e y=11 

Em 31 Aug 2003, [EMAIL PROTECTED] escreveu: 

Marta comprou petecas, bolas e boneca, pagando por 
 cada unidade, respectivamente, 1,00; 10,00; 20,00. 
 Gastou 220,00 em um total de 102 unidades desses 
 brinquedos. Quantas petecas ela comprou? 
 
Seja x o número de petecas 
 y o número de bolas 
 z o número de bonecas 
equacionando devidamente, temos: 
1x+10y+20z=220 
x+y+z=102 
 
olá pessoal, como faço para resolver esta quetão, já q 
nao consigo resolver esta equação? 
 
___ 
Desafio AntiZona: participe do jogo de perguntas e respostas que vai 
dar um Renault Clio, computadores, câmeras digitais, videogames e muito 
mais! www.cade.com.br/antizona 
= 
Instruções para entrar na lista, sair da lista e usar a lista em 
http://www.mat.puc-rio.br/~nicolau/olimp/obm-l.html 
= 
 
-- 

_
Voce quer um iGMail protegido contra vírus e spams? 
Clique aqui: http://www.igmailseguro.ig.com.br
Ofertas imperdíveis! Link: http://www.americanas.com.br/ig/
Ofertas imperdíveis!

=
Instruções para entrar na lista, sair da lista e usar a lista em
http://www.mat.puc-rio.br/~nicolau/olimp/obm-l.html
=


Re: [obm-l] =?Re: [obm-l] duvida de calculo?=

2003-08-20 Por tôpico Fabio Henrique
Se não me enganei com a tua notação, você precisa calcular int[1,x](e^-2s 
ds) 

Fazendo u = -2s temos du = -2ds 
Assim, int[1,x](e^-2s ds) = int[1,x](-1/2.-2.e^-2s ds) = -1/2.int[1,x](e^u 
du) = -1/2.e^u = -1/2.e^(-2s)com s de 1 a x. 
= -1/2.[e^(-2x)-e^(-2). (*) 

Assim F'(x)= 3x^2 . int + x^3 . e^(-2x)este último fator é a derivada do 
resultado (*) 






Em 20 Aug 2003, [EMAIL PROTECTED] escreveu: 

Veja comentários no corpo do texto... 
 
-- Mensagem original -- 
 
Pessoal, por gentileza..me ajudem nisto daqui, pois travei numa parte. 
 
obs: Notacao: Int[1,x] lê-se Integral de 1 até x 
 
Calcule F'(x) sendo F dada por 
F(x) = (x^3).Int[1,x](e^(-s))^2 ds  
 
Minha tentativa de resolucao: 
Seja G uma primitiva da integral, entao 
F(x) = (x^3) (G(x) - G(1)) 
F(x) = (x^3)(G(x)) - (x^3)(G(1)) 
F'(x) = (3x^2)(G(x)) + (x^3)(G'(x)) - 3(x^2)G(1) 
F'(x) = (3x^2)(Int[1,x](e^(-s))^2 ds) + (x^3)((e^(-x))^2 ) - 3(x^2)G(1) 
 
Aqui tem um erro: G(x) não é Int[1,x](e^(-s))^2 ds, mas sim, como você 
mesmo 
definiu, G(x) - G(1) = Int[1,x](e^(-s))^2 ds. Isso resolve o seu problema, 
pois o 3(x^2)G(1) vai cancelar com o G(1) que você esqueceu de subtrair. 
 
 
Nao consigo sair daí...o que é G(1) ??? 
 
A resposta do livro é: 
 
F'(x) = (3x^2)(Int[1,x](e^(-s))^2 ds) + (x^3)((e^(-x))^2 ) 
 
Uma outra maneira de ver isso é usar o Teorema Fundamental do Cálculo e 
dizer (derivada em relação a x) de Int[a, x]f(t) dt = f(x), se f(x) for 
contínua, e então utilizar a regra do produto, o que dá o mesmo resultado 
que acima. 
 
Té mais, 
Bernardo Costa 
 
Obrigado 
 
 
= 
Instruções para entrar na lista, sair da lista e usar a lista em 
http://www.mat.puc-rio.br/~nicolau/olimp/obm-l.html 
= 
 
 
-- 
Use o melhor sistema de busca da Internet 
Radar UOL - http://www.radaruol.com.br 
 
= 
Instruções para entrar na lista, sair da lista e usar a lista em 
http://www.mat.puc-rio.br/~nicolau/olimp/obm-l.html 
= 
 
-- 

_
Voce quer um iGMail protegido contra vírus e spams? 
Clique aqui: http://www.igmailseguro.ig.com.br
Ofertas imperdíveis! Link: http://www.americanas.com.br/ig/
Ofertas imperdíveis!

=
Instruções para entrar na lista, sair da lista e usar a lista em
http://www.mat.puc-rio.br/~nicolau/olimp/obm-l.html
=


Re: [obm-l] ENQUETE - BELEZA MATEMATICA

2003-08-14 Por tôpico Fabio Henrique
A tal beleza começou pela idéia. 
Parabéns. 


Em 09 Aug 2003, [EMAIL PROTECTED] escreveu: 

Caros colegas da lista: 
 
Gostaria de contar com sua participacao numa enquete sobre beleza 
matematica. 
 
O que eu precisao eh que cada um de voces me envie uma lista contendo algo 
como 5 a 10 problemas/teoremas que voces consideram os mais bonitos e cujas 
solucoes/demonstracoes sao as mais elegantes e/ou inusitadas e/ou 
engenhosas. Nao precisa incluir a solucao/demonstracao, apenas o enunciado. 
No entanto, se voce tiver em mente uma solucao/demonstracao especifica 
(entre varias existentes) nao deixe de mencionar pelo menos o metodo 
utilizado. 
 
A unica restricao eh que estes resultados devem ser de um nivel acessivel a 
um aluno normal de 2o. grau (ou seja, o Ultimo Teorema de Fermat e o 
Porisma 
de Poncelet estao fora, mas o caso n = 4 do UTF e a versao para triangulos 
do Porisma poderiam ser incluidos). 
 
Importante: os resultados devem ser acessiveis a um aluno normal de 2o. 
grau, mas nao necessariamente fazer parte do curriculo normal do 2o. grau. 
 
Tambem nao precisa responder hoje ou amanha ou mesmo na semana que vem. 
Acho 
que vale a pena pensar por um tempo e consultar a literatura - as vezes 
pode 
ter um resultado belissimo do qual voce simplesmente se esqueceu por nao 
encontra-lo ha muito tempo. As Eurekas sao uma otima referencia. O Proofs 
from the Book tambem, apesar de nem tudo lah ter nivel de 2o. grau. 
 
Se houver um numero suficiente de respostas, eu me comprometo a publicar 
uma 
compilacao dos problemas e teoremas mais votados. 
 
Desde jah a gradeco o interesse de quem quiser participar. 
 
Um abraco, 
Claudio. 
 
= 
Instruções para entrar na lista, sair da lista e usar a lista em 
http://www.mat.puc-rio.br/~nicolau/olimp/obm-l.html 
= 
 
-- 

_
Voce quer um iGMail protegido contra vírus e spams? 
Clique aqui: http://www.igmailseguro.ig.com.br
Ofertas imperdíveis! Link: http://www.americanas.com.br/ig/
Ofertas imperdíveis!

=
Instruções para entrar na lista, sair da lista e usar a lista em
http://www.mat.puc-rio.br/~nicolau/olimp/obm-l.html
=


Re: [obm-l] Questões_da_ESAEx_Por_favor!

2003-08-03 Por tôpico Fabio Henrique
@ªQuestão: 

lim f(x)/g(x)= lim f'(x)/g'(x)= lim f''(x)/g''(x) 

lim (e^t-cost-sent)/t^2 = lim (e^t+sent-cost)/2t = lim (e^t+cost+sent)/2 

Se t-0, então (e^0+cos0+sen0)/2 = 1 






Em 3 Aug 2003, [EMAIL PROTECTED] escreveu: 

Estou me matando e não consigo! 
 
 1) Somatório de n=1 até n=+oo de 
1/ 
(n+1)^n^n 
? 
 
 2) lim (e^t - cost - 
sent)/t^2 
? 
 t-0 
 
 3) O produto das distâncias de um ponto qualquer de 
uma hipérbole de equação 
 (x^2/a^2) - (y^2/b^2) = 1 às suas 
assíntotas é ? 
 
 4) A derivada n-ésima da função 
 x = ln t , y = t^m , t0 é 
igual a? ACHO QUE FALTAM DADOS tentei fazer com f(x,y) = x.y 
 
 5) Dada a matriz 
 
 | 
-1/2 -5/2 1 
 | 
 
 A 
= | 
-1/2 -1 
1/2 | 
 | 
-3/2 -3 
3/2 | 
 
 podemos afirmar que ( I - A)^ -1 
= 1 + A + A^2 
 
 6) Como resolver o determinante 
abaixo? 
 
 | 
 1 1 
 1 1 
1 
 1 | 
 
| 
a -a 
 b -b 
 c -c 
| 
 | a^2 
a^2 b^2 b^2 c^2 
 c^2 | 
 
| 
a^3 -a^3 
b^3 -b^3 c^3 
-c^3 | 
 | a^4 
a^4 b^4 
b^4 c^4 c^4 
| 
 
| a^5 -a^5 
b^5 -b^5 c^5 
-c^5 | 
 
 7) Considerando um sistema linear de 10 equações e 
10 incógnitas, o número de multiplicações e divisões 
 necessárias para resolvê-lo pela regra de Cramer é 
igual a ? 
 
 8) Se lim [(nx + 1)/(x-1)]^x = 9, qual o valor de n 
? 
 
x- + oo 
 
 9) A matriz da transformação que passa xy = 1 para 
a forma canônica (x^2) / 2 + (y^2)/2 = 1 é ? 
 
 10) Qual o valor de Somatório de p=0 até 
n de 1/(p+1) 
 . Cn,p 
 
 QUEM POR FAVOR PUDER AJUDAR, ESTAS QUESTÕES SÃO DE 
CONCURSOS PASSADOS DA ESAEX! 
 O CONCURSO É EM SET E AINDA ESTOU MUITO 
FRACO! 
 
 forte abraço 
 
 CLEBER 
 
-- 

_
Voce quer um iGMail protegido contra vírus e spams? 
Clique aqui: http://www.igmailseguro.ig.com.br
Ofertas imperdíveis! Link: http://www.americanas.com.br/ig/
Ofertas imperdíveis!

=
Instruções para entrar na lista, sair da lista e usar a lista em
http://www.mat.puc-rio.br/~nicolau/olimp/obm-l.html
=


Re: [obm-l] Questões_da_ESAEx_Por_favor!

2003-08-03 Por tôpico Fabio Henrique
Observe que A^3 = matriz nula (faça as contas) 

Assim, I = I-A^3 
I = I-A^3+(A-A+A^2-A^2) esta parte não altera a igualdade 
I = I+A+A^2-AI-A^2-A^3  troquei um A por AI 
I = I+A+A^2-A(I+A+A^2) 
I = (I+A+A^2)(I-A)  multiplique por (I-A)^-1 pela direita 
(I-A)^-1 = (I+A+A^2) 






Em 3 Aug 2003, [EMAIL PROTECTED] escreveu: 

Estou me matando e não consigo! 
 
 1) Somatório de n=1 até n=+oo de 
1/ 
(n+1)^n^n 
? 
 
 2) lim (e^t - cost - 
sent)/t^2 
? 
 t-0 
 
 3) O produto das distâncias de um ponto qualquer de 
uma hipérbole de equação 
 (x^2/a^2) - (y^2/b^2) = 1 às suas 
assíntotas é ? 
 
 4) A derivada n-ésima da função 
 x = ln t , y = t^m , t0 é 
igual a? ACHO QUE FALTAM DADOS tentei fazer com f(x,y) = x.y 
 
 5) Dada a matriz 
 
 | 
-1/2 -5/2 1 
 | 
 
 A 
= | 
-1/2 -1 
1/2 | 
 | 
-3/2 -3 
3/2 | 
 
 podemos afirmar que ( I - A)^ -1 
= 1 + A + A^2 
 
 6) Como resolver o determinante 
abaixo? 
 
 | 
 1 1 
 1 1 
1 
 1 | 
 
| 
a -a 
 b -b 
 c -c 
| 
 | a^2 
a^2 b^2 b^2 c^2 
 c^2 | 
 
| 
a^3 -a^3 
b^3 -b^3 c^3 
-c^3 | 
 | a^4 
a^4 b^4 
b^4 c^4 c^4 
| 
 
| a^5 -a^5 
b^5 -b^5 c^5 
-c^5 | 
 
 7) Considerando um sistema linear de 10 equações e 
10 incógnitas, o número de multiplicações e divisões 
 necessárias para resolvê-lo pela regra de Cramer é 
igual a ? 
 
 8) Se lim [(nx + 1)/(x-1)]^x = 9, qual o valor de n 
? 
 
x- + oo 
 
 9) A matriz da transformação que passa xy = 1 para 
a forma canônica (x^2) / 2 + (y^2)/2 = 1 é ? 
 
 10) Qual o valor de Somatório de p=0 até 
n de 1/(p+1) 
 . Cn,p 
 
 QUEM POR FAVOR PUDER AJUDAR, ESTAS QUESTÕES SÃO DE 
CONCURSOS PASSADOS DA ESAEX! 
 O CONCURSO É EM SET E AINDA ESTOU MUITO 
FRACO! 
 
 forte abraço 
 
 CLEBER 
 
-- 

_
Voce quer um iGMail protegido contra vírus e spams? 
Clique aqui: http://www.igmailseguro.ig.com.br
Ofertas imperdíveis! Link: http://www.americanas.com.br/ig/
Ofertas imperdíveis!

=
Instruções para entrar na lista, sair da lista e usar a lista em
http://www.mat.puc-rio.br/~nicolau/olimp/obm-l.html
=


Re: [obm-l] Re:_[obm-l]_Questões_da_ESAE x_Por_favor!

2003-08-03 Por tôpico Fabio Henrique
Disponha. 



Em 3 Aug 2003, [EMAIL PROTECTED] escreveu: 

Eu não lembro desse teorema da derivada dos limites de jeito nenhum! 
A questão é muito fácil, vendo por este lado! 
 
Te agradeço mesmo, Fábio, perdi muito tempo nesta questão! 
 
- Original Message - 
From: Fabio Henrique 
To: 
Sent: Sunday, August 03, 2003 3:20 PM 
Subject: Re: [obm-l] Questões_da_ESAEx_Por_favor! 
 
 @ªQuestão: 
 
 lim f(x)/g(x)= lim f'(x)/g'(x)= lim f''(x)/g''(x) 
 
 lim (e^t-cost-sent)/t^2 = lim (e^t+sent-cost)/2t = lim (e^t+cost+sent)/2 
 
 Se t-0, então (e^0+cos0+sen0)/2 = 1 
 
 
= 
Instruções para entrar na lista, sair da lista e usar a lista em 
http://www.mat.puc-rio.br/~nicolau/olimp/obm-l.html 
= 
 
-- 

_
Voce quer um iGMail protegido contra vírus e spams? 
Clique aqui: http://www.igmailseguro.ig.com.br
Ofertas imperdíveis! Link: http://www.americanas.com.br/ig/
Ofertas imperdíveis!

=
Instruções para entrar na lista, sair da lista e usar a lista em
http://www.mat.puc-rio.br/~nicolau/olimp/obm-l.html
=


Re: [obm-l] Re: [obm-l] AJUDA POR FAVOR

2003-08-03 Por tôpico Fabio Henrique
Regra de L'Hôpital 



Em 3 Aug 2003, [EMAIL PROTECTED] escreveu: 

Agradeço! 
Seu resultado bate com o gabarito, mas me surgiu uma dúvida: qual teorema 
que diz que surgindo 
indeterminação podemos derivar que acharemos o mesmo resultado? 
 
Obrigado 
- Original Message - 
From: 
To: 
Sent: Sunday, August 03, 2003 2:39 PM 
Subject: [obm-l] Re: [obm-l] AJUDA POR FAVOR 
 
 
 2) lim (e^t - cost -sent)/t^2? 
 t-0 
 
 Se eu entendi os códigos do enunciado , ai vai : 
 
 Verificamos o caso de indeterminação 0/0 , e por isso , podemos derivar 
 o numerador e o denominador , afim de sumir com o caso de indeterminação 
 : 
 
 [e^t + sent - cost]/2t 
 
 A indeterminação ainda figura na expressão , por isso , repetimos o 
processo 
 : 
 
 [e^t + cost + sent ]/2 
 
 Observe agora que a indeterminação some , quando substituimos t por 0 . 
 
 [1 + 1 + 0]/2 = 1 
 
 então 
 
 lim (e^t - cost -sent)/t^2 = 1 
 t-0 
 
 
 
 
 Tente fazer o outro limite usando algum limite fundamental e pense bem 
nas 
 questões de somatório , são bem legais , vale a pena pensar um pouco mais 
 . 
 
 Abraços 
 
 Luiz H. barbosa 
 
 
 
 www.olympicmaths.hpg.com.br 
 
 
 -- 
 Use o melhor sistema de busca da Internet 
 Radar UOL - http://www.radaruol.com.br 
 
 
 
 = 
 Instruções para entrar na lista, sair da lista e usar a lista em 
 http://www.mat.puc-rio.br/~nicolau/olimp/obm-l.html 
 = 
 
= 
Instruções para entrar na lista, sair da lista e usar a lista em 
http://www.mat.puc-rio.br/~nicolau/olimp/obm-l.html 
= 
 
-- 

_
Voce quer um iGMail protegido contra vírus e spams?
Clique aqui: http://www.igmailseguro.ig.com.br
Ofertas imperdíveis! Link: http://www.americanas.com.br/ig/
Ofertas imperdíveis!

=
Instruções para entrar na lista, sair da lista e usar a lista em
http://www.mat.puc-rio.br/~nicolau/olimp/obm-l.html
=


Re: [obm-l] Re:[obm-l]_estou_com_dúvida. ..

2003-07-22 Por tôpico Fabio Henrique
Você esqueceu que 3-sqrt(17)/-2 é um número positivo. Como esta solução está 
subordinada à hipótese de x0, então deve ser descartada. 



Em 20 Jul 2003, [EMAIL PROTECTED] escreveu: 

 O número de raízes reais distintas da equação x|x|-3x 
 + 2=0 é? 
 
 a)0 
 b)1 
 c)2 
 d)3 
 e)4 
 resolvendo para x0 
x^2-3x+2=0 
x=1 e x=2 
resolvendo para x0 
-x^2-3x+2=0 
x=3+sqrt(17)/-2 
x=3-sqrt(17)/-2 portanto 4 soluções reais distintas 
 ___ 
 
 Yahoo! Mail 
 Mais espaço, mais segurança e gratuito: caixa postal de 
 6MB, antivírus, proteção contra spam. 
 http://br.mail.yahoo.com/ 
 === 
== 
 Instruções para entrar na lista, sair da lista e usar a 
 lista em 
 http://www.mat.puc-rio.br/~nicolau/olimp/obm-l.html 
 === 
== 
 
 
__ 
Acabe com aquelas janelinhas que pulam na sua tela. 
AntiPop-up UOL - É grátis! 
http://antipopup.uol.com.br/ 
 
= 
Instruções para entrar na lista, sair da lista e usar a lista em 
http://www.mat.puc-rio.br/~nicolau/olimp/obm-l.html 
= 
 
-- 

_
Voce quer um iGMail protegido contra vírus e spams? 
Clique aqui: http://www.igmailseguro.ig.com.br
Ofertas imperdíveis! Link: http://www.americanas.com.br/ig/
Ofertas imperdíveis!

=
Instruções para entrar na lista, sair da lista e usar a lista em
http://www.mat.puc-rio.br/~nicolau/olimp/obm-l.html
=


Re: [obm-l] Re:[obm-l]_estou_com_dúvida. ..

2003-07-22 Por tôpico Fabio Henrique
Ignore a minha observação. 



Em 20 Jul 2003, [EMAIL PROTECTED] escreveu: 

 O número de raízes reais distintas da equação x|x|-3x 
 + 2=0 é? 
 
 a)0 
 b)1 
 c)2 
 d)3 
 e)4 
 resolvendo para x0 
x^2-3x+2=0 
x=1 e x=2 
resolvendo para x0 
-x^2-3x+2=0 
x=3+sqrt(17)/-2 
esta não vale por ser positiva 
x=3-sqrt(17)/-2 portanto 3 soluções reais distintas 
 desculpem o erro 
anterior_ 
__ 
 ___ 
 
 Yahoo! Mail 
 Mais espaço, mais segurança e gratuito: caixa postal de 
 6MB, antivírus, proteção contra spam. 
 http://br.mail.yahoo.com/ 
 === 
== 
 Instruções para entrar na lista, sair da lista e usar a 
 lista em 
 http://www.mat.puc-rio.br/~nicolau/olimp/obm-l.html 
 === 
== 
 
 
__ 
Acabe com aquelas janelinhas que pulam na sua tela. 
AntiPop-up UOL - É grátis! 
http://antipopup.uol.com.br/ 
 
= 
Instruções para entrar na lista, sair da lista e usar a lista em 
http://www.mat.puc-rio.br/~nicolau/olimp/obm-l.html 
= 
 
-- 

_
Voce quer um iGMail protegido contra vírus e spams? 
Clique aqui: http://www.igmailseguro.ig.com.br
Ofertas imperdíveis! Link: http://www.americanas.com.br/ig/
Ofertas imperdíveis!

=
Instruções para entrar na lista, sair da lista e usar a lista em
http://www.mat.puc-rio.br/~nicolau/olimp/obm-l.html
=


  1   2   >